Section 9 Flashcards

1
Q

A 35-year-old male with chronic migraine, and taking multiple medications, comes to the clinic with inadequate pain relief. He states that the headaches have been affecting his quality of life. He is recommended to have the placement of an occipital nerve stimulator. He undergoes a trial placement of electrodes, connected to an external battery, and his symptoms are observed for a week. A permanent nerve stimulator is best warranted after which of the following clinical findings?
1. No change in the number of headache days
2. Improvement in quality of life
3. Less than fifty percent improvement of pain
4. Response to conventional drugs

A

2. Improvement in quality of life

  • A trial placement of electrodes is performed under sedation first before permanent occipital nerve stimulator (ONS) implantation.
  • The trial period lasts for 4-7 days. During this period, the patient is required to maintain a headache diary to note the duration and frequency of headaches.
  • It is considered successful if there is more than fifty percent improvement in pain, or there is a decrease in the number of headache days or the patient reports improvement in the quality of life.
  • It helps in the selection of candidates for a permanent stimulator placement.
How well did you know this?
1
Not at all
2
3
4
5
Perfectly
2
Q

A 47-year-old man presents with a one-year history of persistent low back pain. The patient has a past medical history of obesity. On examination, there is paraspinal muscle tenderness. Pain is elicited when the patient is instructed to extend and rotate the spine. A lumbar spine x-ray with flexion and extension shows moderate facet arthropathy in the lower lumbar levels. Which of the following is the most appropriate test to confirm this patient’s most likely diagnosis?
1. MRI of the lumbar spine
2. CT scan of the lumbar spine
3. Diagnostic medial branch block
4. Two diagnostic medial branch blocks

A

4. Two diagnostic medial branch blocks

  • The physical exam is unreliable in diagnosing lumbosacral facet syndrome.
  • The most sensitive test for diagnosing facet syndrome is the medial branch block.
  • There is a high false-positive rate for medial branch blocks.
  • Two blocks should be performed to confirm the diagnosis before proceeding to radiofrequency ablation.
How well did you know this?
1
Not at all
2
3
4
5
Perfectly
3
Q

The embryologic progenitor cell of a chordoma turns into what adult structure?
1. Dura
2. Paraspinous muscle
3. Nucleus pulposus
4. Vertebral body

A

3. Nucleus pulposus

  • Chordomas arise from the notochord.
  • The notochord gives rise to the nucleus pulposus in adults.
  • The notochord is of mesodermal origin and helps signal tissues in the embryo for organization and differentiation.
  • The sclerotomes form the vertebral bodies.
How well did you know this?
1
Not at all
2
3
4
5
Perfectly
4
Q

A 57-year old male noncompliant to medications for diabetes went to his provider due to occasional numbness on both feet for 3 months. Both tuning fork and monofilament tests showed decreased vibration, light touch, and pressure on both feet. Which of the following neuropathway regulates the transmission of pressure and vibration sensory signals?
1. Spinothalamic pathway
2. Corticospinal pathway
3. Posterior column pathway
4. Bulbar pathway

A

3. Posterior column pathway

  • Pain and temperature sensation travels via the spinothalamic tract.
  • Vibration and pressure travels via the posterior columns.
  • The corticospinal tract transmits neural signals to skeletal muscle.
  • Semmes-Weinstein monofilament assesses the protective ability (evaluates A-beta fibers, determining the patient’s threshold for light touch and pressure) of the foot. Tuning fork is used for vibration assessment evaluating the large-diameter fibers (A- beta fibers).
How well did you know this?
1
Not at all
2
3
4
5
Perfectly
5
Q

A 22-year-old football player strikes the ground with his right olecranon process. He leaves the field, and the team physician examines him. The player screams in pain when the physician begins palpating his medial epicondyle and olecranon. The cutaneous nerve fibers responsible for this pain are derived from which of the following components of the brachial plexus?
1. Lateral cord
2. Posterior cord
3. Medial cord
4. Ulnar nerve

A

3. Medial cord

  • The skin of the medial forearm and olecranon receives cutaneous innervation from the medial antebrachial cutaneous nerve.
  • The medial antebrachial cutaneous nerve is a branch of the medial cord of the brachial plexus.
  • The medial cord of the brachial plexus is derived from the inferior trunk of the brachial plexus.
  • The inferior trunk of the brachial plexus is derived from the ventral rami of spinal segments, C8 and T1.
How well did you know this?
1
Not at all
2
3
4
5
Perfectly
6
Q

A 30-year-old male patient presents with complaints of unremitting headaches, dysphagia, shoulder pain, neck pain, stiffness, and weakness. He complains that the pains keep him up at night, and he occasionally has night sweats. On physical examination, the patient appears cachectic, with a BMI of 17.9. On auscultation of the carotid arteries, no bruits are appreciated. On imaging, a mass appears to be occluding an area on the base of the skull lateral to the foramen magnum on the right side. What other clinical feature is most likely to be present in this syndrome?
1. Swaying to the left side while walking
2. Positive Romberg test
3. Deviation of uvula towards the left side
4. Weakness in turning the head to the right side

A

3. Deviation of uvula towards the left side

  • Vernet syndrome is paralysis of cranial nerves IX, X, and XI.
  • The symptoms of the disorder include hoarseness, soft plate drooping, and deviation of uvula towards the normal side. In most cases, the uvula deviates away from the affected side.
  • Other features of the syndrome include loss of gag reflex, dysphagia, and paralysis of the trapezius. The weakness of the sternocleidomastoid muscle (due to spinal accessory nerve palsy) causes difficulty in turning the head to the opposite side.
  • Swaying is a cerebellar sign, and a positive Romberg test is for proprioception, both of which are unlikely to be present in this case. Causes of this syndrome include tumors, trauma, infection, and vascular.
How well did you know this?
1
Not at all
2
3
4
5
Perfectly
7
Q

A 20-year-old man is hit in the head with a baseball bat. He is brought to the emergency department unconscious and displaying signs of decerebrate rigidity. Which of the following would most likely cause an increase in extensor posturing?
1. Damage to the lower motor neurons
2. Chemically stimulating the red nucleus
3. A decrease in neurotransmitters on the gamma motor neurons
4. Destruction of the medullary reticulospinal tract

A

4. Destruction of the medullary reticulospinal tract

  • The vestibulospinal tract and the ponto reticulospinal tract aid in muscle extension.
  • Upper motor neurons synapse onto both alpha and gamma lower motor neurons in the anterior gray horn.
  • The red nucleus is part of the rubrospinal tract, and its activation leads to flexor muscle contraction.
  • A decrease in neurotransmitters in the gamma motor neurons will not lead to increased extensor posturing.
How well did you know this?
1
Not at all
2
3
4
5
Perfectly
8
Q

A patient with a C7 spinal cord injury complains about having a pounding headache. It is noted that he has a flushed, red face. You decide to take his vitals and note that his pulse is 42 beats per minute and his blood pressure is 180/95 mmHg. What should be done immediately?
1. Have the patient lie down and ensure that his abdominal binder is tight
2. Have the patient lie down and loosen his abdominal binder
3. Keep the patient upright, dangling his legs, and give him some aspirin for his headache
4. Keep the patient upright, dangling his legs, and loosen any tight, constricting garments

A

4. Keep the patient upright, dangling his legs, and loosen any tight, constricting garments

  • Any spinal cord injured patient with a T6 or higher lesion is at risk of autonomic dysreflexia. The higher and more complete the injury, the greater the risk. This patient has several cardinal signs of autonomic dysreflexia. His blood pressure is very high, and he is bradycardia while complaining of a severe headache. Lying him down will raise his blood pressure, potentially increasing his risk of a stroke.
  • Loosening the abdominal binder is a good idea. Aspirin is not the right choice for this headache. The clinician needs to determine the cause of the autonomic dysreflexia and correct it immediately, as this is a medical emergency.
  • Autonomic dysreflexia is most often due to an overfull, distended bladder such as from a blocked Foley catheter, so checking for a kinked or blocked urinary catheter is an essential early step in treatment. Asking the patient when they were last catheterized might be helpful, but in an emergency situation dealing with possible autonomic dysreflexia, a Foley catheter should be placed in all susceptible individuals if they don’t already have a urinary drainage catheter. 85% of all cases of autonomic dysreflexia occur due to bladder distension issues. Bowel distension from constipation or fecal impaction is the next most likely cause.
  • If the underlying trigger cannot be determined, emergency antihypertensive pharmacological treatment should be started. Initial treatments include 1-2 inches of 2% nitroglycerine paste or nifedipine 10 mg “bite and swallow.” Sublingual nifedipine is not recommended due to inconsistent absorption. Alternative medical treatment would include sublingual captopril 25 mg or sublingual clonidine 0.2 mg. It is recommended that susceptible patients carry an emergency treatment kit containing critical personal, medical, and contact information. It should also have guidelines on treating autonomic dysreflexia and appropriate supplies, including replacement Foley catheters, anesthetic gel, gloves, a blood pressure cuff (with extra batteries if appropriate), syringes, and normal saline for irrigation. Such a kit is potentially life-saving.
How well did you know this?
1
Not at all
2
3
4
5
Perfectly
9
Q

A 45-year-old man presents with gradual onset deviation of the angle of the mouth to the left side for the past month. He also complains of occasional pain on the right side of the back of his head. On physical examination, the angle of the mouth grossly deviates to the left side, and he cannot close his right eye completely. An MRI brain reveals a lesion arising from the right cerebellopontine angle extending to the supratentorial compartment. He undergoes surgery through the right retrosigmoid corridor. The tentorium is incised to remove the supratentorial component of the tumor. What is the action of the muscle innervated by the nerve most likely to be injured during this procedure?
1. Closure of the right eye
2. Intorsion and depression of the right eye
3. Deviation of the uvula to the right side
4. Deviation of the tongue to the right side

A

2. Intorsion and depression of the right eye

  • This patient has an infratentorial tumor with supratentorial extension, most likely a vestibular schwannoma or cerebellopontine angle meningioma.
  • The nerve likely to be injured during a tentorial incision is the trochlear nerve.
  • The trochlear nerve controls intorsion and depression of the eye by innervating the superior oblique muscle.
  • The trochlear nerve arises from the midbrain below the inferior colliculus. It passes anteriorly between the posterior cerebral and superior cerebellar arteries, enters the wall of the cavernous sinus, and reaches the superior oblique muscle of the eye after exiting through the superior orbital fissure.
How well did you know this?
1
Not at all
2
3
4
5
Perfectly
10
Q

A 16-year-old male patient is brought to the emergency department by his friends. The friends explain that they were at the gym doing some weight training when their friend complained of a headache, but continued to exercise. Over the subsequent hour, his headache worsened, and he had two episodes of vomiting in the gym changing room. On examination, the patient is awake but very confused (E4V4M6). A CT Head is performed and reveals a hyperdense lesion, with surrounding hypodensity, in the left frontal lobe with mild mass effect. The lateral ventricles, the third ventricle, and the fourth ventricle are enlarged. The neurosurgery team is informed, and they are on their way to the emergency department to assess the patient. While doing neuro-observations, the nurse notices the patient’s right pupil is larger than the left, and it is not reactive. The neurosurgery team is beeped while they are in the stairwell, and they ask that the patient be immediately prepped for emergency surgery, and IV mannitol started. In the emergency setting, what single surgical intervention is indicated for this patient?
1. Decompressive craniectomy without evacuation of the intracerebral hemorrhage (ICH)
2. CSF diversion in the form of an external ventricular drain (EVD)
3. Posterior fossa craniectomy
4. Catheter aspiration of the ICH (minimally invasive surgery)

A

2. CSF diversion in the form of an external ventricular drain (EVD)

  • A ‘fixed and dilated pupil’ in the context of intracranial pathology is a sign of raised ICP, and this needs management in a time- critical manner. Medical intervention includes positioning the patient head-up, hyperventilation to maintain a pCO2 between 4.5-5.5kPa, and giving an osmotic agent such as mannitol or hypertonic saline. The enlarged ventricles in this setting indicate hydrocephalus, and the emergency neurosurgical procedure to relieve this would be CSF diversion in the form of an external ventricular drain (EVD).
  • In a young patient with an ICH, it is important to rule out an underlying pathology (i.e. vascular malformation) before surgery in order to assist in surgical planning.
  • Posterior fossa decompression can be a life-saving treatment in infratentorial ICH; however, it is not an appropriate surgical procedure for decompression of a supratentorial lesion.
  • A decompressive craniectomy may be an appropriate surgical intervention in an attempt to save a patient’s life; however, in the setting of acute hydrocephalus, CSF drainage is the priority.
How well did you know this?
1
Not at all
2
3
4
5
Perfectly
11
Q

A 35-year-old male patient presents to the emergency department via emergency medical service (EMS) following a motor vehicle collision that occurred on a local thoroughfare. The patient was the unrestrained driver. Vitals show temperature 37, blood pressure 148/76 mmHg, pulse rate: 140 beats per minute, respiratory rate: 26 breath per minute, saturation 97% on room air. The primary survey shows airway, circulation, and breathing intact. GCS is 14 for confusion at the time of examination. Head, eye, ear, nose, and throat exam (HEENT) exam is notable for bilateral retro auricular ecchymosis. Visual field testing shows a loss of peripheral vision bilaterally. Where is the pathologic lesion most likely located?
1. Optic nerve
2. Optic radiations
3. Optic chiasm
4. Optic tract

A

3. Optic chiasm

  • Traumatic fracture of the frontal bone or anterior base of the skull may be due to the lesion of the optic chiasm producing bitemporal hemianopsia. In this patient, retro auricular ecchymosis, otherwise known as the battle sign, is suggestive of a basilar skull fracture.
  • Bitemporal hemianopsia is a result of damage to the optic chiasm, specifically the medial nasal fibers which decussate at this location.
  • The most appropriate diagnostic modality for imaging the sellar region is MRI.
  • Damage to the optic radiations may produce a contralateral inferior or superior quadrantanopia, depending on whether the lesion affects the parietal or temporal areas, respectively. Lesions of the optic nerve produce monocular blindness in the affected eye. Homonymous hemianopsia, in conjunction with an afferent pupillary defect, localizes the lesion to the optic tract.
How well did you know this?
1
Not at all
2
3
4
5
Perfectly
12
Q

A patient develops meningitis after head trauma. The patient expires and the autopsy reveals softening of brain tissue and diffuse edema. The patient most likely had what type of necrosis?
1. Coagulation necrosis
2. Liquefactive necrosis
3. Wedge infarct
4. Fat necrosis

A

2. Liquefactive necrosis

  • Liquefactive necrosis is transformation of solid tissue into fluid.
  • Liquefactive necrosis is typical of bacterial or fungal infections.
  • The necrosis is due to a release of hydrolytic enzymes.
  • Liquefactive necrosis often results in abscess formation.
How well did you know this?
1
Not at all
2
3
4
5
Perfectly
13
Q

A 48-year-old man presents to the emergency department after eight days of continuous headaches. On the first day, he had a severe sudden headache, but he thought it was related to his migraine history. When he was born, a myelomeningocele was repaired on his first day of life but did not require a ventricular shunt. A head computed tomographic (CT) scan with and without contrast does not show evidence of blood or a tumor. No evident aneurysm is seen. A cervical puncture is performed as the lumbar area cannot be punctured because of the history of myelomeningocele. Which of the following areas of the brainstem is most at risk of penetration due to its proximity to the puncture site during the procedure in this patient?
1. Cisterna magna
2. Medulla oblongata
3. Cerebellum
4. Jugular bulb

A

2. Medulla oblongata

  • The cervical puncture is potentially dangerous because the puncture needle directly faces the medulla oblongata. The needle should only be advanced 1-2 mm after the dura is entered.
  • The cisterna magna is located between the cerebellum and medulla oblongata, and the needle should enter this space to obtain cerebrospinal fluid.
  • The medulla oblongata is responsible for autonomic functions of breathing, heart rate, and blood pressure, as well as the sleep- wake cycle. If the needle enters into it, the patient may develop acute changes in heart rate and pulse.
  • The jugular bulb is not at risk as it is inside the bone of the mastoid and temporo-occipital bone.
How well did you know this?
1
Not at all
2
3
4
5
Perfectly
14
Q

How can a patient with neurogenic claudication improve his symptoms?
1. Lying prone
2. Walking straight
3. Walking downhill
4. Riding bicycle

A

4. Riding bicycle

  • The pain of spinal canal stenosis is relieved by methods that help to open up the inter-laminar distance and thereby open up the canal diameter.
  • This helps to relieve the root as well as cord compression as well as improve their blood circulation.
  • The cycling that requires stooping posture helps to open up increase the cord diameter thereby helping to relieve the symptoms of lumbar canal stenosis.
  • Walking increases the pain of neurological claudication. Walking downhill requires a lumbar extension that furthers compromises the canal diameter thereby aggravating the symptomatology.
How well did you know this?
1
Not at all
2
3
4
5
Perfectly
15
Q

A 2-month-old boy is being evaluated for flattening of the left posterior head. He has a preferred sleeping position. His forehead is prominent on the left side. The view from above shows the shape of the head as a parallelogram. CT head shows patent sutures. What is the most appropriate management strategy for this patient?
1. Endoscopic surgery and helmet
2. Open surgery and reconstruction
3. Sleep positional change and helmet
4. Mannitol

A

3. Sleep positional change and helmet

  • The diagnosis is positional plagiocephaly because of the parallelogram shape of the head and the patent sutures in CT.
  • Only conservative treatment is needed in positional plagiocephaly.
  • Change of sleeping position to either side in supine will reduce the progression of plagiocephaly.
  • Helmet therapy is for molding of the head in the case of positional plagiocephaly.
How well did you know this?
1
Not at all
2
3
4
5
Perfectly
16
Q

Neurosurgical techniques like prefrontal leucotomy and transorbital leucotomy were erroneously used during the early periods of the 20th century to treat psychiatric disorders and in most cases causing more harm than good and leading to stigma towards surgical approaches in treating psychiatric disorders. Which of the following is not one of the rare advances that have been made that can help reduce the stigma towards neurosurgical techniques in treating psychiatric disorders?
1. The development of numerous modalities that helped in the understanding of the structure and function of the brain
2. Positive results in the use of neuromodulation surgery like deep brain stimulation in the treatment movement disorders like parkinsonian diseases
3. The possibility of using neuromodulation to modify the cognition of healthy individuals
4. The development of advanced stereotactical microsurgery techniques

A

3. The possibility of using neuromodulation to modify the cognition of healthy individuals

  • The arrival of new diagnostic techniques and the ability to integrate information from different diagnostic modalities has made it possible for clinicians to understand the brain structure and function and help localize brain pathologies. Some of these diagnostic techniques include functional magnetic resonance imaging (fMRI), positive emission tomography (PET), neurophysiologic data from electroencephalogram (EEG), magnetoencephalography (MEG), transcranial magnetic stimulation (TMS). All these diagnostic modalities provide more in-depth knowledge of brain activities and a stronger foundation for specific targets for neuromodulation surgery as a treatment for some psychiatric disorders.
  • While the stigma due to the history psychosurgery still looms, the utilization of neuromodulatory surgery to treat psychiatric disorder is backed by not only a more profound understanding of brain structure and function, but also, the development of new stereotactic microsurgical technique, and data about successful management of other neurological disorders through neuromodulation. This fountain of knowledge has led to the development of treatment options that target a particular region of the brain and with effects of these treatments that can be reversible, unlike the earlier process like prefrontal leucotomy that involves the permanent removal of parts of the brain. For example, the concept of deep brain stimulation in the treatment of psychiatric disorders can be attributed to DBS success in the treatment of movement disorder like Parkinson disease, essential tremor, and dystonia.
  • In addition to the old concerns, there is a new wave concern as to what the limits of what some of these neuromodulatory surgical procedures will be. Professionals debate whether these techniques will be used only as a last resort for the treatment of refractory psychiatric symptoms, or if they will be used for other purposes like to modify the cognition of healthy individuals.
  • There have been many advances made in stereotactical microsurgery and also the tools available, decreasing the complications that may arise from these procedures.
How well did you know this?
1
Not at all
2
3
4
5
Perfectly
17
Q

A 3-year-old male child presented with seizures. His mother claims that he stands independently but needs to hold on to something to walk; he can grasp a pencil and do random scribbles, follow 1 step commands, and knows 6 to 10 words. he is fully dependent when eating and dressing. On examination, there is spasticity on the left upper and lower extremities compared to the right. Babinski was positive bilaterally. Imaging studies show a cleft in the right cerebral hemisphere. Which of the following lines is the cleft in the brain?
1. CSF
2. Gray matter
3. Hamartomatous tissue
4. White matter

A

2. Gray matter

  • Schizencephaly clefts are lined by gray matter of either the heterotopic, pachy- or polymicrogyric variety.
  • The clefs extend from the pial surface lateral to the ependymal lateral ventricular lining medially.
  • Large primitive veins may overlie the cleft.
  • Porencephaly also has clefts due to trauma or vascular insult, and these clefts are thus lined by gliotic white matter, not dysplastic gray matter.
How well did you know this?
1
Not at all
2
3
4
5
Perfectly
18
Q

A 45-year-old female presents with progressive headaches and multiple episodes of vomiting. Fundoscopy reveals bilateral papilledema. A CT of the head demonstrates a large, hyperdense lesion in the left lateral ventricle with evidence of hydrocephalus. MRI of the brain shows a lesion with a soap-bubble appearance, and MR spectroscopy demonstrates choline and glycine peaks. Which of the following is the most likely lesion?
1. Intraventricular abscess
2. Central neurocytoma
3. Intraventricular tuberculoma
4. Glioblastoma

A

2. Central neurocytoma

  • A hyperdense intraventricular lesion causing hydrocephalus that demonstrates a soap-bubble appearance on MRI is characteristic of a central neurocytoma.
  • Choline and glycine peaks on MR spectroscopy are commonly seen in central neurocytoma and are also seen in other higher- grade intracranial tumors. The rapid turnover of cells in malignancy releases choline from the phosphatidylcholine component of the cell membrane, leading to its characteristic peak in a spectroscopy sequence.
  • Patients presenting with headaches and vomiting should undergo urgent or emergent head CT to identify intracranial causes, such as obstructive hydrocephalus.
  • A tuberculoma will show an increase in the lipid peak.
How well did you know this?
1
Not at all
2
3
4
5
Perfectly
19
Q

A 16-year-old patient is brought to an emergency department following a motor vehicle collision. He is complaining of persistent neck pain without any radiation. He has no motor or sensory deficits. CT spine revealed the presence of bilateral fractures of the C2 pars interarticularis with a displacement of 1.5 mm without any angulation. What is the most rational management strategy for the patient?
1. Hard Cervical collar
2. Soft cervical collar
3. Halo-vest orthosis
4. C1-C3 posterior fusion

A

1. Hard Cervical collar

  • Hangman’s fracture is a fracture of the pedicles or pars interarticularis of the C2 vertebra.
  • Nonunion is rare in stable hangman’s fractures, with approximately 90% healing with immobilization alone. Level III evidence shows that a hangman’s fracture may be initially managed with immobilization with a halo-vest or collar alone.
  • External orthosis should be maintained for 8 to 14 weeks. This produces a reduction rate of 97% to 100% and a fusion rate of 93% to 100%.
  • It is important to remember that halo-vest orthosis is not very well tolerated and has poor patient compliance, and therefore collar is recommended as first-line management.
How well did you know this?
1
Not at all
2
3
4
5
Perfectly
20
Q

A 65-year-old man fell from the roof of his barn. EMS is notified, and he is transported to the nearest emergency department. Upon arrival, he is noted to be comatose, with a Glasgow coma scale of 6/15. A noncontrast CT scan of the head demonstrates a depressed skull fracture in the right occiput. A right parietal-occipital subdural hematoma is also noted. After initial stabilization, he is transferred to the closest level 1 trauma center. He undergoes craniotomy for elevation of his depressed skull fracture and evacuation of the subdural hematoma. However, his neurological exam does not improve. His post-operative course is complicated by intermittent episodes of intracranial hypertension, which respond to treatment with hypertonic saline. By hospital day 7, his intracranial pressure is well-controlled, and he is no longer receiving sedation or osmotherapy. He remains intubated. Brainstem reflexes are intact. He opens his eyes to noxious stimulation, but he does not follow commands. He localizes to pain with the right upper extremity and withdraws from noxious stimulation in the left upper extremity. A brain MRI demonstrates bifrontal hemorrhagic contusions as well as scattered punctate areas with susceptibility artifact in the corpus callosum and bilateral subcortical white matter. However, none were noted in the brainstem. Median nerve somatosensory evoked potentials are present bilaterally, and EEG is notable for a breach rhythm in the right parietooccipital region with bifrontal focal slowing but no epileptiform discharges. Which of the following best describes the patient’s long-term prognosis?
1. The patient has sustained a severe traumatic brain injury, and it will likely be fatal
2. The patient has sustained a moderate traumatic brain injury; with continued supportive care and intensive rehabilitation, a full recovery is likely
3. The patient has sustained a severe traumatic brain injury; even with continued supportive care, it is unlikely that he will return to his prior level of function
4. The patient has sustained a severe traumatic brain injury, and tests results suggest there may be a cervical spine injury as well

A

3. The patient has sustained a severe traumatic brain injury; even with continued supportive care, it is unlikely that he will return to his prior level of function

  • The patient sustained a severe traumatic brain injury resulting in bifrontal contusions, a depressed skull fracture, and a right- sided subdural hematoma. His exam did not improve notably after surgical intervention, and magnetic imaging findings were consistent with diffuse axonal injury (DAI). In patients with DAI, neurological dysfunction occurs secondary to axonal disruption. It is possible for this dysfunction to improve over time due to axonal regrowth, although recovery is slow.
  • In this example, the patient has an asymmetric motor exam on hospital day 7, as well as significant cognitive dysfunction, which is apparent due to his depressed level of consciousness. Exam and EEG findings are consistent with the structural lesions seen on MRI, including bilateral frontal contusions. Pathologically, the cerebral contusions result in cellular necrosis leading to irreversible neuronal loss in the affected brain regions. Consequently, some degree of permanent disability will be likely.
  • Upon initial presentation in the vignette above, the patient had a GCS of 6, consistent with a diagnosis of severe traumatic brain injury. The exam described on hospital day 7 is suggestive of a GCS of 8t. In order to appropriately classify traumatic brain injury, the GCS should be performed on admission, but after appropriate resuscitation has been performed.
  • Somatosensory evoked potentials (SSEPs) can be useful for determining prognosis in survivors of traumatic brain injury. Bilateral absence of the cortical response after median nerve stimulation is associated with a universally poor outcome. However, it is important to exclude other injuries along the neuraxis in order to avoid a false-positive test. For example, lesions of the brachial plexus or the cervical spine can confound the result. In this patient, the median nerve SSEPs are present bilaterally, which is not suggestive of a cervical spine or brachial plexus lesion. Unfortunately, preserved SSEP responses do not reliably predict neurological recovery.
How well did you know this?
1
Not at all
2
3
4
5
Perfectly
21
Q

A 48-year-old man presents with contusions throughout the cervical, thoracic, and lumbar regions following a motor vehicle accident. His vitals are stable, and a computerized tomography (CT) scan of the head is negative for acute pathology, but on physical examination, there is a left-sided miosis associated with ptosis. A CT scan of the neck, thorax, and abdomen is ordered. At which of the following levels is he likely to have a lesion?
1. C3
2. C7
3. T3
4. L4

A

3. T3

  • Horner syndrome is a condition in which damage to the sympathetic nervous system leads to miosis, ptosis, anhidrosis, and sometimes enophthalmos.
  • The sympathetic nervous system exits the spinal cord from T1- L2/L3.
  • Horner syndrome can be caused by damage to the sympathetic nervous system at various points in the sympathetic tract; it commonly results from an infarct in the lateral medulla, damage during thyroid resection, impingement by Pancoast tumor, or spinal cord injury.
  • Horner syndrome is not dangerous in itself but is often a clinical indicator for one of the conditions mentioned above that may be deadly.
How well did you know this?
1
Not at all
2
3
4
5
Perfectly
22
Q

A 32-year-old woman with multiple sclerosis is evaluated in the clinic for sensory deficits. She is currently on glatiramer acetate and interferon therapy. She explains that she can feel, describe, and touch items in her hand but has trouble identifying the object. When asked about the object or when seeing the object, the woman is able to identify the object. Where is the most likely location of the lesion?
1. Primary somatosensory cortex
2. Secondary somatosensory cortex
3. Spinothalamic tract
4. Pacinian corpuscles

A

2. Secondary somatosensory cortex

  • The woman’s multiple sclerosis is an autoimmune insult to the nervous system that has caused her to lose the ability to recognize sensory stimuli. A lesion to the secondary somatosensory cortex, or association cortex, can result in an inability to recognize objects that can be felt. This is called astereognosis or tactile agnosia.
  • The secondary somatosensory cortex has connections to the hippocampus and amygdala, allowing us to relate the tactile sensation to prior exposure.
  • In this woman, her ability to describe the item in her hand is intact, while her ability to assess the tactile stimulus is impaired. Furthermore, she is able to visually identify objects that she cannot identify using touch, indicating that her memory or attention is likely not impaired.
  • Lesions in the spinothalamic tract and Pacinian corpuscles results in pain disruption. Lesions to the primary somatosensory cortex can result in an inability to recognize objects that were felt but can also affect the ability to assess features of the object itself, such as texture, shape, and size.
How well did you know this?
1
Not at all
2
3
4
5
Perfectly
23
Q

A 16-year-old male patient has been brought to the emergency department following a gunshot injury at the medial epicondyle of his right elbow joint. During the surgical exploration of the wound, the ulnar nerve was found to be completely scattered by the injury. During the follow-up visits, there was a sensory loss in his fourth and fifth fingers of the right hand. There was a minimal weakness of the intrinsic muscles of the hand without any claw deformity. What is the anatomical basis for these paradoxical clinical findings seen in this patient with ulnar nerve injury?
1. Martin-Gruber anastomosis
2. Marinacci syndrome
3. Riches-Cannieu communication
4. Berrettini branch

A

1. Martin-Gruber anastomosis

  • The Martin–Gruber anastomosis is a forearm communication between the median and the ulnar nerve, with a prevalence of almost 60%.
  • This communication is the commonest form of motor communication between the median and the ulnar nerve. The type 1 variant is the commonest form of this communication in which the fascicles from the anterior interosseous nerve join the median nerve.
  • This communication helps to preserve the motor function of the intrinsic muscles of the hand supplied by the ulnar nerve, despite its injury. This knowledge is of paramount importance in understanding the perplexing neurological findings in such anatomical variants.
  • Marinacci syndrome is the ulnar to median nerve communication in the forearm. The Berrettini branch is the superficial sensory communication between the ulnar and the median nerve in the palm of the hand.
How well did you know this?
1
Not at all
2
3
4
5
Perfectly
24
Q

An 8-year-old boy falls while playing soccerball. Because a readily performed radiological examination reveals a wrist fracture, the child undergoes a surgical treatment. A few days after the operation, the little patient reports neurological symptomatology featuring in numbness and tingling sensation over the lateral 3/4th of the right palm and fingers. What could be the cause of this neurological symptomatology?
1. Ulnar nerve injury
2. Median nerve injury
3. Radial nerve injury
4. Axillary nerve injury

A

2. Median nerve injury

  • Median nerve injury could result from compartment syndrome or carpal tunnel syndrome (CTS).
  • Overuse of the hands can cause CTS and rest is often advised.
  • When the median nerve is compressed at the wrist, the individual will have weakness in abduction and opposition of the thumb.
  • The paresthesias may occur in the lateral 3 1/2 digits including the nailbeds. Initially, the treatment of carpal tunnel syndrome is to stop the activity causing symptoms and wearing a splint.
How well did you know this?
1
Not at all
2
3
4
5
Perfectly
25
Q

A 45-year-old patient presents with persistent headaches, which are bitemporal, nonprogressive, and aggravated towards the end of the day. There is no aura, photophobia, or phonophobia. The patient has no red eyes or running nose during the episodes. CT head is normal, and erythrocyte sedimentation rate (ESR) is normal. What is the early recommended modality in the management of the headache in the patient?
1. Injection at the trigger points
2. Biopsy of the superficial temporal artery
3. Electrical stimulation
4. Massage of the temporalis muscle

A

4. Massage of the temporalis muscle

  • The patient has signs and symptoms consistent with the diagnosis of tension headache.
  • The presence of trigger points, which are formed following idiopathic sustained but a focal contraction of the muscle leading to ischemia, is the cause of the headache.
  • Manual massage helps to improve the blood supply, thereby reducing the pain originating from the trigger points.
  • The patient has normal ESR, so the likely diagnosis of superficial temporal arteritis is minimal. The redness of the eyes and running nose are associated with cluster headaches. Injections at the trigger points are undertaken in cases of refractory and persistent headaches.
How well did you know this?
1
Not at all
2
3
4
5
Perfectly
26
Q

A 60-year-old man is brought to the emergency department with complaints of confusion and headache for the past three days. He is a smoker and has a history of type 2 diabetes mellitus. On examination, his Glasgow coma scale score is E3V4M5, and his right pupil is bigger than his left. He undergoes a computed tomogram of the head, which shows a space-occupying lesion in his right temporal lobe, which is mildly enhancing in the periphery. There is a midline shift of 9 mm to the left. He undergoes an emergency craniotomy and decompression of the lesion. The histopathology report shows that it is a glioma showing nuclear atypia, the presence of cellular pleomorphism, and marked mitotic activity with the absence of necrosis and endovascular proliferation. Which of the following is the most likely diagnosis?
1. WHO Grade I astrocytoma
2. WHO Grade II astrocytoma
3. WHO Grade III astrocytoma
4. Gemistocytic astrocytoma

A

3. WHO Grade III astrocytoma

  • WHO Grade III astrocytomas (also known as anaplastic astrocytomas) infiltrate the surrounding brain parenchyma diffusely and have an intrinsic tendency for malignant progression to glioblastoma. These tumors may also have regions of lower-grade tumors intermixed.
  • Histological examination shows higher cellular and nuclear atypia than seen in grade II astrocytoma, but there is an absence of necrosis and microvascular proliferation seen in glioblastoma.
  • Molecular profiling reveals a heterogeneous genetic mutation profile within the grade of astrocytoma. Prognosis varies widely depending on the genetic makeup between tumors of similar histological grade. Patients with unfavorable molecular patterns have a prognosis similar to grade 4 tumors (glioblastoma multiforme).
  • IDH (isocitrate dehydrogenase) mutations present in grade 3 astrocytoma portend a favorable prognosis compared to IDH wild type genotypes.
How well did you know this?
1
Not at all
2
3
4
5
Perfectly
27
Q

A 67-year-old man presents to the clinic with a chief complaint of chronic axial low back pain. His pain is persistent, non-radiating, and described as dull and achy in nature. During the physical exam, the clinician asks the patient to bend backward while standing. As the patient did this, he reports increased pain in his low back. Which of the following is the most reliable method to diagnose the condition observed in the patient?
1. Dynamic x-ray lumbosacral spine
2. CT lumbosacral spine
3. MRI lumbosacral spine
4. Facetal block

A

4. Facetal block

  • A positive Kemp’s test in the patient indicates facet-mediated pain. Another name for Kemp’s test is the quadrant test.
  • Kemp’s test is performed in the lumbar spine by having the patient extend the spine with associated rotation. Kemp’s test will be positive if the patient’s baseline pain is reproduced while performing the maneuver.
  • The diagnostic ultrasound or fluoroscopic guided facetal block is the most reliable means for diagnosing facet-mediated pain, with level I or level II evidence based on the United States Preventive Services Task Force criteria.
  • The dynamic X-ray is indicated in spinal instability. Radiological imagings only provide supportive evidence in facetal joint arthropathy.
How well did you know this?
1
Not at all
2
3
4
5
Perfectly
28
Q

A 65-year-old female becomes suddenly unresponsive and is taken to the emergency room where workup reveals subarachnoid hemorrhage. A posterior communicating artery aneurysm is identified and treated. One day after presentation, she is alert and interactive but having severe headaches. Five days after admission, she is witnessed to have a seizure. Her workup identifies serum sodium of 122 meq/L, urine sodium of 55 meq/L, urine osmolality of 111 mosmol/kg, and hypovolemia. What is the most appropriate treatment?
1. Hypertonic fluids and careful monitoring of serum sodium
2. Fluid restriction and careful monitoring of serum sodium
3. Thiazide diuretic and careful monitoring of serum sodium
4. Foley catheterization

A

1. Hypertonic fluids and careful monitoring of serum sodium

  • Cerebral salt wasting and syndrome of inappropriate anti- diuretic hormone (SIADH) can be confused as lab results are similar in both.
  • For cerebral salt wasting a patient is hypovolemic whereas in SIADH they are euvolemic to hypervolemic.
  • Treatment for cerebral salt wasting includes restoring fluid volume status and replacing lost sodium.
  • For SIADH treatment includes fluid restriction.
How well did you know this?
1
Not at all
2
3
4
5
Perfectly
29
Q

A 40-year-old woman who recently had an epidural steroid injection two days ago presents to the clinic complaining of an occipital headache every time she tries to get to an upright position. The patient states that she started having the headache a few hours after the procedure. Her neurological exam is normal. What is the next best step in the management of this patient?
1. MRI brain w/o contrast
2. MRI lumbar spine
3. Advice increased fluid and caffeine intake
4. Perform an epidural blood patch

A

3. Advice increased fluid and caffeine intake

  • The patient’s symptoms and timing of onset of headache are consistent with spontaneous intracranial hypotension.
  • This a common complication after epidural steroid injections and can happen with an inadvertent dural puncture resulting in leakage of CSF.
  • This results in low intracranial pressure causing occipital headaches. The common finding is the worsening of headaches in an upright position.
  • Any further imaging is not indicated at this time since there are no changes in her neurological exam. An epidural blood patch is reserved for patients that have failed conservative therapy, including high volume fluids and caffeine. In most cases, the condition usually resolves with conservative management.
How well did you know this?
1
Not at all
2
3
4
5
Perfectly
30
Q

A 2-year-old boy is brought to the clinic for a well- child exam. The mother received no antenatal care, and he was delivered by a spontaneous vaginal delivery at 36 weeks gestation. He has no apparent anomalies. Examination reveals an ocular coloboma. No other physical defects are evident. Family history reveals neonatal death in a paternal cousin after they detected an intracranial malformation. Which of the following findings is most likely to be seen on a brain MRI in this patient?
1. Midline monoventricle
2. Midline and anterior cerebral arteries
3. Absent olfactory bulb
4. Fused deep grey nuclei

A

2. Midline and anterior cerebral arteries

  • This infant with an ocular coloboma and no other anomaly is most likely a case of a middle intrahemispheric variant. An MRI would reveal bilateral frontal and parietal lobes, an absent body of the corpus callosum, and azygous anterior cerebral artery are visible in these patients.
  • Holoprosencephaly (HPE) results from an incomplete midline cleavage of the forebrain (prosencephalon) and includes a wide spectrum of intracranial and craniofacial midline defects, along with a myriad of clinical manifestations, which consist of neurologic impairment and dysmorphism of the brain and face.
  • Holoprosencephaly occurs rather frequently, and is observed in 1:250 conceptuses; due to a high rate of fetal demise, the birth prevalence is between 1:8000 to 1:16000 live births and is similar among different international populations. HPE is associated with pregestational maternal diabetes. Consistent maternal folic acid use appears to be protective.
  • Midline monoventricle, fused deep grey nuclei, and absent olfactory bulbs are features of alobar and semi lobar variant. These are associated with facial anomalies like pronounced microcephaly cyclopia, cebocephaly, ethmocephaly.
How well did you know this?
1
Not at all
2
3
4
5
Perfectly
31
Q

A 54-year-old woman fell off a ladder at home and presents with neck pain. X-rays reveal a loss of cervical lordosis in the lower cervical spine, and CT confirms a C6/7 unilateral facet fracture-dislocation. Which of the following sets of physical examination findings is most likely to be seen in this patient?
1. Unilateral weakness in wrist extension, with numbness and tingling in the small finger
2. Bilateral weakness in elbow flexion (palm up), with numbness and tingling in the thumb
3. Unilateral weakness in elbow extension, with numbness and tingling in the small finger
4. Unilateral weakness in shoulder abduction, with numbness and tingling in the lateral forearm

A

3. Unilateral weakness in elbow extension, with numbness and tingling in the small finger

  • Proper x-rays of the cervical spine must include AP, lateral, and open mouth odontoid films. Films must contain the entire cervical spine and include C7-T1. On the lateral x-ray, you must examine the 4 parallel lines of the cervical spine. Signs of cervical facet dislocation can include vertebral body subluxation compared to the vertebral body below. Unilateral facet dislocation can lead to about 25% subluxation, bilateral facet dislocation can lead to about 50% subluxation. Loss of disc height might indicate a retropulsed disc in the canal.
  • Due to the pedicle nerve root mismatch of the cervical spine, the nerve root exits above the corresponding pedicle. Therefore, a C6-C7 dislocation would cause impingement on the C7 nerve root.
  • Injury to the C7 nerve root can present with deficits in elbow extension and wrist flexion, as it primarily innervates the triceps and flexor carpi radialis—also, sensory deficits in fingers 2, 3, 4, and an abnormal triceps reflex.
  • Typically, unilateral facet dislocations lead to unilateral symptoms. Unilateral weakness in wrist extension is associated with impingement/injury to the C6 nerve root. Unilateral weakness in shoulder abduction, with numbness and tingling in the lateral arm is associated with impingement/injury to the C5 nerve root.
How well did you know this?
1
Not at all
2
3
4
5
Perfectly
32
Q

A neonate is delivered and found to have a small 1.2 cm anterior skin-covered mass just above the nasal bridge at the glabella. APGAR 9/10. Eye position is normal, and respirations are adequate. Head circumference is 50% for age. The anterior fontanelle is open and easily depressible. Ultrasound confirms the sac is filled with fluid. The 10th-week fetal ultrasound did not show the mass. In which area of the head did this developmental anomaly occur?
1. Craniopharyngeal canal
2. Foramen cecum
3. Cribriform plate
4. Ethmoid bone

A

2. Foramen cecum

  • The patient described has an anterior nasofrontal encephalocele. Anterior encephalocele development occurs from abnormal development of the foramen cecum. A diverticulum of the dura usually projects anteriorly between the developing nasal and frontal bones at the fonticulus frontalis/foramen cecum.
  • Later in embryogenesis, the diverticulum regress and the bone closes; however, if it does not regress, the brain can herniate through the bone defect and form an encephalocele.
  • Sincipital encephaloceles are classified as nasofrontal, nasoethmoidal, or naso-orbital. Nasofrontal encephalocele is the most common type, seen in 46.4% of the patients. It is followed by nasoethmoidal type in 39.2% of the patients. The naso-orbital and the combined type are the least common with 14.2%.
  • Nasofrontal encephaloceles result from herniation through the foramen cecum and the fonticulus frontalis and project along the nasal bridge between the nasofrontal sutures into the glabella. Nasoethmoidal encephaloceles occur when there is herniation through the foramen cecum into the prenasal space and nasal cavity under the nasal bones and above the nasal septum. Naso-orbital encephaloceles occur along the medial orbit wall at the level of the frontal process of the maxilla and the ethmoid- lacrimal bone junction.
How well did you know this?
1
Not at all
2
3
4
5
Perfectly
33
Q

A 78-year-old female presents to the clinic with numbness and tingling in her left foot. She has a history of chronic low back pain which she aggravated after she lifted a heavy box while twisting. She also has a history of posterior knee pain from a Baker’s cyst. Which of the following muscles is crucial in distinguishing between an L5 radiculopathy and a tibial neuropathy on electromyography (EMG) in this patient?
1. Peroneus longus muscle
2. Medial gastrocnemius muscle
3. Lateral gastrocnemius muscle
4. Tibialis posterior muscle

A

1. Peroneus longus muscle

  • On electromyography, it is key to test muscles innervated by different peripheral nerves but the same nerve root. The peroneus longus is innervated by the superficial peroneal nerve, predominantly from the L5 nerve root. Testing this muscle and comparing the results to other L5 and tibial nerve innervated muscles can help distinguish L5 radiculopathy from tibial neuropathy.
  • The medial gastrocnemius muscle is innervated by the tibial nerve, by both the L5 and S1 nerve roots. However, it is generally predominantly from the S1 nerve root which makes this a common muscle to test on EMG to rule out S1 radiculopathy due to its easy accessibility.
  • The lateral gastrocnemius muscle is innervated by the tibial nerve, by both the L5 and S1 nerve roots. However, it is generally predominantly from the L5 nerve root. Testing this muscle would not be very helpful in distinguishing a L5 radiculopathy from tibial neuropathy due to the overlap in its innervation.
  • The tibialis posterior muscle is innervated by the tibial nerve, by both the L5 and S1 nerve roots. However, it is generally predominantly from the L5 nerve root. Testing this muscle would not be very helpful in distinguishing a L5 radiculopathy from tibial neuropathy due to the overlap in its innervation.
How well did you know this?
1
Not at all
2
3
4
5
Perfectly
34
Q

A 32-year-old male patient presents with complaints of intermittent episodes of headache, dysphagia, torticollis, hoarseness of voice, and pulsatile tinnitus. Clinical examination reveals paralysis of sternomastoid, loss of gag reflex, and vocal cord palsy on the right side. The CT scan of the head shows a 3x4 cm sized mass with a moth-eaten appearance in the posterior cranial fossa, and an MRI scan shows salt-and-pepper appearance in both long TR and short TR images. What is the next best step in the management of this patient?
1. Observation
2. Pre-operative embolization followed by safe surgical excision
3. Stereotactic radiosurgery
4. Fractionated stereotactic radiotherapy

A

2. Pre-operative embolization followed by safe surgical excision

  • The diagnosis is paraganglioma (glomus jugular).
  • Pre-operative embolization, followed by safe surgical excision is the treatment of choice.
  • Stereotactic radiosurgery (Gamma Knife, LINAC, and CyberKnife) is a useful and safe treatment option for paragangliomas at this location, if the diameter is less than 3 cm, in order to reduce the morbidity.
  • Fractionated stereotactic radiotherapy is the technology which combines the precision of stereotaxy with dose fractionation in large tumors (more than 3–4 cm in diameter) which are not suitable for radiosurgery.
How well did you know this?
1
Not at all
2
3
4
5
Perfectly
35
Q

A 40-year-old man is scheduled for a spinal procedure for disc herniation. He is on long-term anticoagulant therapy due to the replacement of the aortic valve ten years back. Physical examination is unremarkable. Vital signs are a pulse of 75/min, a blood pressure of 122/78 mmHg, a respiratory rate of 17/min, and a temperature of 98.6 F (37 C). Which of the following should be performed before the procedure?
1. Continue the same dose of anticoagulant therapy
2. Reduce the dose of anticoagulant therapy
3. Stopping the anticoagulant therapy
4. Increase the dose of anticoagulant therapy

A

3. Stopping the anticoagulant therapy

  • Warfarin is an oral anticoagulant commonly used to treat and prevent blood clots. It is given in case of mechanical valve repair.
  • In a high-risk procedure, it is recommended to stop warfarin five days before and start bridging therapy with low molecular weight heparin. The heparin should be stopped 24 hours before in a high-risk procedure.
  • Warfarin inhibits the hepatic synthesis of coagulation factors II, VII, IX, and X, and coagulation regulatory factors protein C and protein S require the presence of vitamin K. Vitamin K is an essential cofactor for the synthesis of all of these vitamin K- dependent clotting factors.
  • The use of warfarin during a high-risk procedure can lead to uncontrolled bleeding and subsequently increase morbidity and mortality.
How well did you know this?
1
Not at all
2
3
4
5
Perfectly
36
Q

A 20-year-old woman, G2P1, who is at 24 wks gestation, came for a regular antenatal checkup. The woman admits to drinking alcohol before she knew she was pregnant. An ultrasound was recommended for the evaluation of the fetus, which showed findings concerning clefts. Further imaging was recommended for evaluation. Which imaging modality would be the diagnostic choice of imaging in this situation?
1. Computed tomography (CT) scan
2. Magnetic Resonance Imaging (MRI)
3. X-ray
4. Repeat two-dimensional ultrasonography

A

2. Magnetic Resonance Imaging (MRI)

  • Magnetic resonance imaging (MRI) is unique to the diagnosis of children with congenital brain abnormalities, and in utero, MR (iuMR) imaging is now used for the antenatal detection of brain abnormalities, including schizencephaly.
  • Various definitions are put forward. One definition of schizencephaly is a trans-mantle column of dysplastic grey matter extending from the ependyma to the pia without a cerebrospinal fluid cleft.
  • Computed tomography may also be useful, but it provides poorer images of the gray matter, which is the key factor in differentiating between schizencephaly and other fluid- associated CNS abnormalities.
  • Schizencephaly may be diagnosed prenatally or postnatally by ultrasonography, but this is true for type II (open-lip) only. The diagnosis of schizencephaly may be suspected prenatally if clefts are viewed within the cerebral hemispheres by two- dimensional ultrasonography (2DUS).
How well did you know this?
1
Not at all
2
3
4
5
Perfectly
37
Q

A 30-year-old female presents for her yearly examination. She states that she and her husband have been trying to conceive for eight months without progress and that her menstrual cycles have become irregular. Her clinician suggests that she and her husband continue trying to conceive and that she return in 4 months for some laboratory studies if she still has not become pregnant. In between, a routine visit to the ophthalmologist shows bitemporal hemianopsia. Which of the following is the most likely cause of infertility in this patient?
1. Endometriosis
2. Failure of implantation
3. Hostile cervical mucus
4. Suppression of gonadotropins and ovulation

A

4. Suppression of gonadotropins and ovulation

  • This patient has a pituitary prolactinoma, which is associated with amenorrhea, infertility, and galactorrhea. Prolactin inhibits the secretion of gonadotropins and suppresses ovulation. The hypothalamus has a predominant inhibitory influence on prolactin secretion via dopamine and any factor which disrupts this mechanism can cause hyperprolactinemia. It is important to consider the various causes of hyperprolactinemia as increased prolactin secretion is noted in many physiological and pathological states other than prolactinomas.
  • Prolactinomas cause a wide range of symptoms either due to the mass effect of the tumor or from the hypersecretion of prolactin. Based on the size of the tumors, prolactinomas can be classified as micro prolactinomas (smaller than 10 mm), macroprolactinomas (larger than 10 mm), or giant prolactinomas (larger than 4 cm). Hyperprolactinemia is not always due to a prolactinoma, and other causes like pregnancy, medications, hypothyroidism and pituitary stalk effects due to other pituitary tumors should be considered in the differential.
  • Unlike other pituitary tumors, the preferred treatment for prolactinomas is medical therapy. Oral contraceptives alone can be given if the only symptoms are amenorrhea and/or osteoporosis. Specific treatment for prolactinomas is one of the dopamine agonists such as bromocriptine. Adverse effects of bromocriptine include nausea, nasal congestion, and orthostatic hypotension. If the side effects are not well tolerated, it can be administered intravaginally.
  • Ectopic endometrial tissue is seen with endometriosis. Prolactin inhibits the secretion of gonadotropins which suppresses ovulation. The ovaries are still responsive to gonadotropins.
How well did you know this?
1
Not at all
2
3
4
5
Perfectly
38
Q

A 3900-gram (85th percentile) newborn male is delivered at 39 weeks gestation to a 30-year-old G3P3 woman. The patient was born via normal vaginal delivery but required vacuum- assisted delivery. At 6 hours of age, the nurse reports a swollen deformity of the baby’s scalp. Upon physical examination, the baby is hemodynamically stable and well-appearing. Scalp examination reveals a soft palpable swelling over the left temporoparietal area, slight discoloration, and it does not cross the suture lines. The patient’s physical examination is otherwise unremarkable. His head circumference is noted to be 36 cm (90th percentile). Which of the following is the next best step in the management of this patient?
1. Incision and drainage
2. CT head
3. Reassurance
4. Ultrasound head

A

3. Reassurance

  • Due to the circumstances of delivery and clinical features (location of the hematoma that does not extend suture lines), the most likely cause of this patient’s condition is a cephalohematoma.
  • Whenever a clinician diagnoses a cephalohematoma, reassurance with counseling of the parents is necessary.
  • Cephalohematomas require observation and most cases resolve in several weeks without any intervention.
  • There is no need to drain a cephalohematoma. The patient’s condition is not consistent with intracranial hemorrhage, as cephalohematomas occur beneath the periosteum. Thus, further imaging is not necessary. If there is a concern for intracranial injury, computed tomography of the brain (if the patient has clinical signs or symptoms of increased intracranial pressure) or magnetic resonance imaging (to avoid ionizing radiation if the patient is stable) is recommended not only to characterize the area but to determine if there is any underlying skull deformity as well (i.e., fracture).
How well did you know this?
1
Not at all
2
3
4
5
Perfectly
39
Q

A 78-year-old male with a past medical history of atrial fibrillation on anticoagulation, insulin-dependent diabetes, and spinal stenosis of the cervical spine at C5-C6 presents to the office with chronic, worsening right arm pain, numbness, tingling, and weakness. Symptoms have been occurring for years but have gotten progressively worse over the last few weeks. He now has difficulty gripping objects with his right hand. Conservative management has failed previously, including physical therapy and oxycodone for pain relief. The symptoms have made it difficult to perform his activities of daily living. His physical exam is significant for a strength of 3 out of 5 on the right in arm’s flexion, the sensation is reduced along the C5 dermatome on the right, and C5 deep tendon reflexes is 1/4. Which of the following is an absolute contraindication of the non-surgical management of this patient?
1. Failure of medical management and physical therapy
2. Progressively worsening neurological symptoms
3. History of symptomatic spinal stenosis of the cervical spine
4. Chronic pain affecting his activity of daily living

A

2. Progressively worsening neurological symptoms

  • Cervical epidural corticosteroid injections would be the next step in management for cervical radiculitis with failed conservative management. However, it is contraindicated in severe, progressive neuropathy, suggestive of a need for surgical correction.
  • The corticosteroid epidural injection is preferred over surgery in patients with progressive pain without worsening neurological findings. Before the procedure, the provider should monitor for progression of symptoms, including associated worsening weakness or sensory changes.
  • In cases of worsening or severe symptoms, a neurosurgeon should be consulted for surgical decompression of the spinal cord rather than a minimally invasive epidural injection.
  • Absolute contraindications for cervical spine epidural injections include worsening and progressing neurological deterioration, but not his history of chronic stenosis of the cervical spine. This alone would not be a contraindication. If his pain were stable, this would be an indication of a cervical spine epidural.
How well did you know this?
1
Not at all
2
3
4
5
Perfectly
40
Q

A patient presents with weakness and cranial nerve deficits. Imaging studies done shows a small unilateral lesion on the basal portion of the ventrocaudal pons. Which of the following structures are expected to be involved in this syndrome?
1. Ipsilateral oculomotor and trochlear nerve, contralateral corticospinal tract
2. Ipsilateral trochlear nerve and abducens nerve, contralateral corticospinal tract
3. Ipsilateral oculomotor nerve and abducens nerve, contralateral corticospinal tract
4. Ipsilateral abducens nerve and facial nerve, contralateral corticospinal tract

A

4. Ipsilateral abducens nerve and facial nerve, contralateral corticospinal tract

  • This patient has Millard-Gubler syndrome (MGS), also known as facial abducens hemiplegia syndrome or the ventral pontine syndrome.
  • MGS is characterized by a unilateral lesion of the basal portion of the ventrocaudal pons involving fascicles of abducens (VI) and the facial (VII) cranial nerve, and the pyramidal tract fibers.
  • This syndrome involves the fibers of cranial nerves VI (abducens), VII (facial) on the ipsilateral side, and corticospinal tract fibers contralaterally. Symptoms include ipsilateral lateral rectus palsy leading to diplopia that is accentuated when the patient looks towards the side of the lesion, internal strabismus (i.e., Esotropia) and loss of power to rotate the affected eye outward due to the involvement of CN VI, ipsilateral peripheral facial nerve paresis leads to flaccid paralysis of the muscles of the facial expression and loss of the corneal reflex due to cranial nerve VII involvement, and contralateral hemiplegia of the extremities (sparing the face) due to pyramidal tract involvement.
  • Neurological imaging, i.e., computed tomography (CT) and magnetic resonance imaging (MRI) help identify the lesion. However, MRI of the brain is more sensitive and specific than CT scan to determine the infarcts at an early stage of onset, especially in the setting of small pontine lesions.
How well did you know this?
1
Not at all
2
3
4
5
Perfectly
41
Q

A 65-year-old male presents to the clinic with pain in his low back and buttocks for the past seven months. He has a history of esophageal reflux, diabetes, and hypertension, for which he takes omeprazole, metformin, and amlodipine. The patient reports he works as a construction worker and that the pain is worse when he is walking around but improves when he sits down and bends forward. The patient denies any trauma to the area. On physical exam, pain with lumbar extension is elicited. The sensation is intact to light touch throughout, and strength is 5/5 in bilateral lower extremities. On nerve conduction studies and needle electromyography, SNAP amplitudes are normal, CMAP amplitudes are normal, and abnormal spontaneous potentials are seen on needle EMG. What is the most likely cause of these findings?
1. Hypertrophic changes to the ligamentum flavum
2. Elevated blood pressure
3. Spondylosis
4. Piriformis syndrome

A

3. Spondylosis

  • The patient in the vignette has spinal stenosis. There are many causes of spinal stenosis. Spinal stenosis is a condition often seen in the geriatric population, and it can be congenital, or, more frequently, acquired.
  • Some examples include spondylolisthesis, intervertebral disc herniation, enlargement of the soft tissues in and around the canal, hypertrophy of the facet joints, or ligamentum flavum hypertrophy or laxity. Such changes can lead to isolated denervations at the paraspinal.
  • Studies have reported an incidence of 1 case per 100,000 for cervical spine stenosis and 5 cases per 100,000 for lumbar spine stenosis.
  • Elevated blood pressure can result in a serious complication known as aortic dissection. This usually occurs at the inner layer of the aorta. This would result in acute severe back pain. In piriformis syndrome, the patient complains of buttocks pain that may or may not travel down the leg. It occurs because the sciatic nerve is irritated. It may worsen after sitting for a long time, climbing stairs, walking, or running.
42
Q

A 26-year-old female presents to the emergency department with a fever and headache. She states she has been sick for the last week. On exam, the patient appears uncomfortable but not toxic. She is currently afebrile. She has mild nuchal rigidity and no rash. Non-contrast CT of the brain shows no acute process and no midline shift or mass. A lumbar puncture is performed. The white blood cell count in the cerebrospinal fluid is mildly elevated with lymphocyte predominance, the protein is elevated, and the glucose is low. A magnetic resonance imaging of the brain reveals basal exudates. What is the most common neurologic manifestation of the pathogen likely to be causing her condition?
1. Meningitis
2. Brain abscess
3. Acute cerebrovascular accident
4. Cranial nerve palsies

A

1. Meningitis

  • Tuberculosis meningitis (TBM) is a common presentation of TB intracranially.
  • In TBM, the meninges are seeded by miliary tuberculosis and form subependymal collections called Rich foci. These foci can rupture into the subarachnoid space and cause an intense inflammatory response that causes the symptoms of meningitis.
  • TBM is also often characterized by marked basal meningitis, as opposed to bacterial meningitis, which tends to produce a meningeal reaction over the convexities of the brain. Other differentials for basal meningitis include Syphilis and low-grade chronic granulomatous inflammation
  • 1% of all cases of TBM present with extra-pulmonary TB. In the developed world, where there is a lower prevalence of TB in the population, it is estimated that TBM accounts for 6% of all causes of meningitis.
43
Q

An 8-year-old girl with severe gelastic seizures undergoes brain biopsy. H&E staining reveals clusters of small neurons intermixed with glia in a grape-like configuration. Patient history includes difficulty in concentration, short term memory, and reduced processing speed for the past year. Which of the following is the next best step in the management of this patient?
1. Antiepileptic drugs
2. Reassurance
3. Surgical resection
4. Thermal ablation

A

3. Surgical resection

  • Surgical resection provides the best treatment method for seizure control, nearly up to 90%.
  • Gelastic seizures are the most common clinical manifestation of hypothalamic hamartomas.
  • Hypothalamic hamartomas can also present with short term memory difficulty, concentration problems, and reduced processing speed.
  • Hypothalamic hamartomas are grouped in clusters and configured in a grape-like arrangement when observed under a hematoxylin and eosin stain.
44
Q

A 68-year-old male with a history of diabetes and hypertension presents with sudden drooping of his left eye, left facial numbness, ataxic gait, and right arm numbness. His vitals are normal. What is the most likely diagnosis?
1. Anterior cerebral stroke
2. Middle cerebral stroke
3. Posterior cerebral stroke
4. Vertebrobasilar stroke

A

4. Vertebrobasilar stroke

  • Wallenberg syndrome, lateral medullary syndrome, is an infarct of the vertebrobasilar or posterior inferior cerebellar arteries and presents with ipsilateral loss of pain and temperature on the face from the descending trigeminal tract, contralateral loss of these sensations on the body, ataxia, and often an ipsilateral Horner syndrome.
  • The vertebrobasilar arterial system provides oxygenated blood to the cerebellum, medulla, pons, thalamus, midbrain, and occipital cortex. The occlusion of these vessels is a significant cause of morbidity.
  • Distinct syndromes of the vertebrobasilar system include the presence of cerebellar signs in most cases (ataxia and dysmetria).
  • If the nuclei of the cranial nerves are affected, the clinical features are on the ipsilateral side and the corticospinal signs are on the opposite leg and arm. Most patients will have dysphagia and dysarthria.
45
Q

A 5-year-old boy is being evaluated for intellectual disability and seizures. On magnetic resonance imaging (MRI) of the brain, the patient is found to have maldevelopment involving the largest white matter structure. Which of the following is the most likely associated feature to be seen in the MRI?
1. Empty sella
2. Frontal lobe atrophy
3. Chiari I malformation
4. Abnormally shaped third and lateral ventricles

A

4. Abnormally shaped third and lateral ventricles

  • With agenesis of the corpus callosum, the third ventricle may be dilated and open to the surface of the brain while the lateral ventricles characteristically have a bat-wing appearance on coronal sections.
  • Clinical symptoms of agenesis of the corpus callosum can range from none to a variety of cognitive limitations.
  • Agenesis of the corpus callosum can present as an isolated anomaly or in association with other congenital anomalies.
  • Agenesis of the corpus callosum with intellectual disability, seizures, chorioretinal lesions, and vertebral anomalies define Aicardi syndrome.
46
Q

A 53-year-old female with breast carcinoma HER 2+ presents with complaints of sudden seizures in the emergency. She has complained of mild headaches and nausea for the last three weeks. She has been treated with a bilateral mastectomy and chemotherapy drugs, including carboplatin, cyclophosphamide, and doxorubicin. On examination, the patient is drowsy and disorientated but able to follow commands. The patient has left facial nerve palsy, left VI palsy, and asymmetric weakness of lower limbs. Power is 4/5, and the Babinski sign is positive bilaterally. Magnetic resonance imaging brain is done; it shows leptomeningeal enhancement, enlarged lateral ventricles, and dilatation of temporal horns bilaterally. CSF cytology shows HER 2+ tumor cells. Which is the best treatment approach?
1. Surgical excision of metastatic lesion
2. Whole-brain irradiation therapy
3. Intrathecal chemotherapy with systemic trastuzumab
4. Systemic chemotherapy with intrathecal trastuzumab

A

4. Systemic chemotherapy with intrathecal trastuzumab

  • Primary cancers causing metastatic infiltration of leptomeningeal layers are variable. They include extra-cranial and intra-cranial sites. Breast carcinoma is one of the most frequently sold cancer to cause carcinomatous meningitis. It occurs in the advanced stage of an aggressive tumor. So, invasive lobular breast carcinoma is the most common among breast cancers to cause this, particularly HER 2+ type. It can present with a wide variety of neurological symptoms like headaches, seizures, cranial nerve symptoms, spinal radiculopathy, cauda equina syndrome, and even like a stroke.
  • The diagnosis is by cerebrospinal fluid studies and magnetic resonance imaging. Cerebrospinal fluid cytology is of vital importance in solid tumors. It holds diagnostic value as well as prognostic value. Different tumor markers can be checked, which can lead to early diagnosis. Magnetic resonance imaging study shows hydrocephalus, leptomeningeal enhancement along sulci and gyri, sub-ependymal nodules, and dilatation of the Virchow-robin spaces in carcinomatous meningitis.
  • Carcinomatous meningitis from breast carcinoma is an aggressive metastatic complication with a dismal prognosis. Management of carcinomatous meningitis due to breast cancer is complex and depends on whether the patient is at high risk or low risk. Good prognostic factors include the Karnofsky performance status scale (KPS) score >60, minimal systemic disease, minimal neurological deficit, low CSF protein, and good treatment options. Poor risk patients are those with KPS score 60, neurological deficits, high systemic disease burden, normal CSF protein, and poor treatment options. Patients with low-risk disease are treated with a combination of intrathecal and systemic chemotherapy, while those with high-risk disease are managed with palliative care.
  • Surgical treatment is rarely done in carcinomatous meningitis. In CNS lesion, which is single and accessible, can be removed with surgery, but in carcinomatous meningitis, only surgery is done for the Omaya catheter placement. Radiotherapy includes focal and whole-brain irradiation therapy. Focal radiotherapy is done for symptomatic management of nodular lesions, while whole-brain radiation therapy is done for linear lesions causing symptoms. Asymptomatic and symptomatic lesions in low-risk disease are treated with a combination of intrathecal and systemic chemotherapy according to ESMO-EANO guidelines. Intrathecal trastuzumab in patients with human epidermal growth factor receptor 2 positive (HER 2+) breast cancer is proving to be an effective therapy.
47
Q

A 60-year-old male complains of a severe, acute- onset headache that started three hours ago. He also complains of nausea that started one hour ago. He denies vomiting, syncope, or muscle weakness. The patient’s past medical history is significant for hypertension, diabetes mellitus type 2, and moderate-severity tension headache. He says this headache is different in location and severity from his usual headaches. His medications include hydrochlorothiazide and metformin. On examination, the temperature is 36.5 C, blood pressure is 161/92 mmHg, and pulse is 84 beats/min. The patient is in significant distress due to pain. He has right-sided ptosis, and his right pupil is larger than his left pupil. There is neck rigidity and loss of sensation in both of his feet. There is no ataxia or motor weakness. Deep tendon reflexes are normal, and Babinski reflex is flexor. Which of the following is most likely causing this patient’s symptoms?
1. Subarachnoid hemorrhage due to posterior communicating artery aneurysm
2. Subarachnoid hemorrhage due to anterior communicating artery aneurysm
3. Diabetes mellitus
4. Subarachnoid hemorrhage due to posterior inferior cerebellar artery aneurysm

A

1. Subarachnoid hemorrhage due to posterior communicating artery aneurysm

  • This patient is complaining of acute onset, severe headache that differs from his usual headaches. The headache is accompanied by right eye ptosis, right pupil dilation, and nuchal rigidity. This constellation of symptoms is worrisome for a subarachnoid hemorrhage due to a posterior communicating artery aneurysm.
  • Cranial nerve three palsy is a typical finding with posterior communicating artery aneurysm due to the close proximity of the nerve to the artery. The aneurysm compresses against the third cranial nerve leading to ptosis and pupil dilation.
  • Subarachnoid hemorrhage usually presents with sudden onset of a severe headache, which can be accompanied by nausea, vomiting, loss of consciousness, focal neurological deficits, and neck rigidity. The headache is usually described as the “worst headache of my life” and is usually in a different location and more severe than the patient’s usual headaches.
  • The loss of sensation in his feet is likely due to diabetes mellitus and not an aneurysm compressing the anterior cerebral artery. This patient should get a non-contrast CT scan, as it is the preferred initial test to establish the diagnosis. A head CT scan has nearly 100% sensitivity for detecting subarachnoid hemorrhages if done within 6-12 hours of the onset of the pain.
48
Q

A 30-year-old man is brought to the emergency department after he was hit by a bike while crossing the road. On examination, his Glasgow coma scale score is 7 (E1V1M5), and his pupils are bilaterally equal and reactive to light. There are no signs of injury to other systems. A computed tomogram scan of the brain is obtained, which shows a temporal contusion (8 ml in volume) on the left side and diffuse brain edema. An intracranial pressure bolt is placed to monitor his intracranial pressure, which shows a reading of 14 mm Hg. After two hours, the nurse notes that the intracranial pressure is 35 mm Hg. Which of the following additional clinical finding is highly consistent in this patient at this point in time?
1. Blood pressure 90/50 mm Hg
2. Heart rate of 45 beats/minute
3. Heart rate of 120 beats/minute
4. Respiratory rate of 30 breaths/minute

A

2. Heart rate of 45 beats/minute

  • According to the Monro-Kellie principle, since the skull is a rigid structure and the skull contents of the brain, cerebrospinal fluid (CSF), and blood are incompressible, an increase in any one or several of these will cause ICP elevation. Irregular respiration, bradycardia, and increased blood pressure result from high intracranial pressure (ICP) and are termed the Cushing response or triad. Irregular respirations are due to hypoperfusion of the brainstem. The Cushing response is a critical event related to head trauma and usually indicates that herniation is imminent.
  • Hypertension is a compensatory response to decreased cerebral perfusion, which is the difference between MAP and ICP. Blood pressure will rise due to vasoconstriction and increased cardiac output to increase CPP.
  • Progressive hypertension with bradycardia and decreased respiratory effort is a specific response to the acute rise in ICP. Cushing’s reflex indicates that the ICP has reached a life- threatening level and herniation is imminent. However, the full triad of hypertension, bradycardia, and respiratory irregularity manifests in only one-third of patients who have life-threatening increased ICP.
  • In head trauma patients with a GCS of 8 or less, an ICP monitor should be placed to monitor intracranial pressure. An increase in ICP is associated with a negative outcome. The “gold standard” for ICP measurement is ventriculostomy.
49
Q

A 65-year-old male with a history of diabetes, hypertension, and chronic atrial fibrillation presents with acute onset of vertigo, dysarthria, dysphagia, and sensory abnormalities on the face and the opposite trunk and extremities. The neurological exam reveals loss of pain and temperature sensation on the same side of the face with decreased pain and temperature sensation on the opposite side of the trunk and extremities. MRI of the brain shows occlusion of the right posterior inferior cerebellar artery. The patient shows improvement during hospitalization. What is the most important treatment for secondary prevention in this patient that has the most significant impact on his subsequent risk of stroke?
1. Smoking cessation
2. Good control of glycated hemoglobin
3. Oral warfarin
4. Combined aspirin and clopidogrel

A

3. Oral warfarin

  • Wallenberg syndrome is due to a cerebrovascular accident (CVA) of the posterior inferior cerebellar artery (PICA).
  • There is sensory, pain, and temperature loss on the face ipsilateral to the lesion and contralateral trunk and extremity involvement. Additional symptoms include ataxia, facial pain, nystagmus, and Horner syndrome.
  • The recurrent stroke risk in this patient is well over 10%, given his multiple risk factors and atrial fibrillation. The stroke prevention in atrial fibrillation (SPAF) trial showed that using oral anticoagulation with warfarin and a target INR of 2 to 3 reduces his risk by two-thirds. All other risk reductions are important, yet the most significant risk here is atrial fibrillation.
  • Combination antiplatelets in the short term (up to 1 month) is better than single antiplatelet, yet oral warfarin is still the best treatment in the presence of atrial fibrillation.
50
Q

A 54-year-old retired female construction worker presents to the clinic for evaluation. Her primary complaint is lower back pain. She mentions pain in her lower back and hip with prolonged standing and walking, which is improved with sitting. Her pain is intermittent, non-radiating, and described as burning, with no lower extremity numbness or tingling. She denies any recent weight loss, fevers, or chills. She denies bowel or bladder incontinence. Her motor strength is 5/5 in both upper and lower extremities. She brings a lumbar MRI from 5 years ago, which shows mild degenerative changes, and is currently taking gabapentin 600 mg twice daily. Which of the following is the next best step in the management of this patient?
1. Pericapsular nerve block
2. Lumbar epidural corticosteroid injection
3. Repeat lumbar MRI
4. Surgery

A

3. Repeat lumbar MRI

  • The history and physical exam are consistent with lumbar spinal stenosis, for which epidural corticosteroid injection typically provides significant relief.
  • The pericapsular nerve block has no role in managing spinal stenosis. The PENG block is ideal for acute pain management for pathology involving the hip joint and upper part of the femur.
  • New imaging needs to be obtained. The patient’s symptoms have worsened, and five-year-old imaging is insufficient to guide treatment.
  • Surgery would be too early at this point. The patient has no motor weakness, no alarm symptoms, and no new imaging suggesting that surgical intervention is necessary at this point. More conservative therapy should be initiated early on, such as physical therapy, medications, and epidural steroid injections.
51
Q

A patient presents to the clinic with pain in upper limbs and head. He tells that he has suffered from this for quite some time now and has not improved with medication. The clinician decides to proceed with the stellate ganglion block using ultrasound for guidance. What is the preferred vertebral level for injecting the local anesthetic for this purpose?
1. C6
2. C7
3. C5
4. T1

A

1. C6

  • The injection is suggested not inferior to the C6 level because the vertebral artery is left unprotected at the C7 level due to its absent or rudimentary anterior tubercle.
  • The stellate ganglion is located at C7 to T1 levels, but the drug is injected int0 the sympathetic chain at the level of C6 from where it spreads along prevertebral fascia to reach the stellate ganglion.
  • Use an in-plane approach to avoid parenchymal and vascular punctures.
  • A color doppler should always be used.
52
Q

A 35-year-old male was brought to the emergency department (ED) after he had a fall from the second floor of his house. He lost consciousness immediately after the fall but regained consciousness en route to the ED. On examination, there was a 3 cm laceration on the left parietal region. He was drowsy but easily arousable and was able to follow commands. A head CT scan was requested, which reveals a lentiform hyperdensity on the left parietal convexity with a 4 mm midline shift. Which of the following neurologic signs should be monitored most importantly in this patient?
1. Progressive weakness on the contralateral side
2. Progressive weakness on the ipsilateral side
3. Pupillary dilatation on the contralateral side
4. Pupillary dilatation on the ipsilateral side

A

4. Pupillary dilatation on the ipsilateral side

  • Pupillary dilatation on the ipsilateral side is an early sign of uncal herniation. In patients with head trauma and deterioration in sensorium or loss of consciousness, a CT scan is indicated.
  • In a patient with increased intracranial pressure due to bleeding or edema, the pressure on one side can push the brain structures past the midline (falcine herniation) and downward (uncal herniation).
  • The downward pressure of the uncus can compress the 3rd cranial nerve, which is in near proximity.
  • This would cause an initial pupillary dilatation on the ipsilateral side, followed by extraocular muscle paresis. Though this lesion may also cause weakness of the contralateral side, the most important parameter to monitor is the pupil size of a drowsy patient.
53
Q

A 54-year-old woman undergoes a left transforaminal epidural steroid injection (TFESI) at C5-6 level for radicular pain. Ten minutes later, the patient develops left arm weakness and bilateral lower extremity weakness. Twenty-four hours later, an MRI is done that shows a patchy increased signal from the odontoid to the C5 level within the cord. There is a gradual recovery of function afterward. What drug was the most likely injected, and where was it injected?

  1. Ropivacaine into the subarachnoid space
  2. Contrast into the subdural space
  3. Dexamethasone into the nerve root
  4. Methylprednisolone into the left vertebral artery
A

4. Methylprednisolone into the left vertebral artery

  • Injection of methylprednisolone (a particulate steroid) into the left vertebral artery caused an embolic event leading to the spinal cord’s infarction.
  • The spinal cord receives blood supply via the vertebral arteries and branches, which are the anterior and posterior spinal arteries.
  • The vertebral arteries are vulnerable structures that may be inadvertently pierced in a transforaminal approach in the cervical spine.
  • Dexamethasone is a non-particulate steroid that has not been associated with embolic complications
54
Q

A 60-year-old woman presents to the outpatient department with complaints of blurring of her right eye vision, a subjective pulsatile orbital bruit, mild protrusion of the eye, and headache over the past one month. She reveals that she had been treated for a head injury six months back. On examination, her visual acuity is 20/70 in the right eye, and there is mild proptosis. She undergoes a computed tomogram angiography of the brain, which shows an enlarged superior ophthalmic vein and a bulging cavernous sinus on the right side with abnormal enhancement of the right cavernous sinus. She is being planned for an interventional radiology procedure involving direct cannulation of the superior ophthalmic vein (SOV). This procedure aims to take direct advantage of SOV to which venous structure?

  1. Inferior petrosal sinus
  2. Cavernous sinus
  3. Facial vein
  4. Supraorbital vein
A

2. Cavernous sinus

  • The superior ophthalmic vein (SOV) ends in the cavernous sinus, making it a frequent target for open transvenous approaches to the site when other access routes have failed.
  • The inferior petrosal sinus is commonly accessed through the jugular vein. This is the preferred endovascular approach to the cavernous sinus when available. It is not traversed in the alternate route via the superior ophthalmic vein.
  • The facial vein is the access point for the standard endovascular approach to the superior ophthalmic vein. However, it may be challenging to use in particular cases due to tortuosity or discontinuity.
  • The supraorbital, angular, and supratrochlear veins feed into the superior ophthalmic vein and may be used as anatomic landmarks during the surgical approach. However, they are not traversed by the catheter.
55
Q

An 8-year-old girl with right-sided hemiplegic cerebral palsy secondary to left-sided cortical malformation presents for follow-up. Previous workups have revealed intractable focal, atonic, and absence seizures. A recent EEG is consistent with electrical status epilepticus in sleep (ESES). Her seizures remain uncontrolled even after trials of six different antiepileptics in multiple combinations. What is the best next step in the management of this patient?

  1. Ketogenic diet
  2. Modified hemispherectomy
  3. Corpus callosotomy
  4. Multiple subpial transections
A

2. Modified hemispherectomy

  • A subgroup of electrical status epilepticus in sleep (ESES) patients with structural brain pathology, such as perinatal infarction or cortical malformation, may benefit from epilepsy surgery.
  • Children with ESES on EEG and unilateral brain pathology are frequently seizure-free after hemispherectomy, and it has been shown to improve cognitive status.
  • Other surgical options like corpus callosotomy and multiple subpial transections are palliative surgeries, and patients are less likely to be seizure-free than with hemispherectomy.
  • A ketogenic diet has been studied in small numbers of patients, and no specific recommendation can be made regarding its efficacy.
56
Q

A 65-year-old woman presents to the emergency department after sustaining a ground-level fall in her home. She reports midback pain. She has normal vital signs, a body mass index (BMI) of 27 kg/m^2, and no other injuries from the fall. Imaging of her spine is obtained with demonstrates 50% collapse of a midthoracic vertebral body with a breach of the anterior cortex alone. What is the appropriate activity level and treatment?

  1. Pain medications and activity as tolerated
  2. Bedrest for 3 weeks to allow the fracture to heal
  3. Fitting of a brace for stabilization then ambulation
  4. Strict spinal precautions until surgical stabilization is performed
A

3. Fitting of a brace for stabilization then ambulation

  • Based on the description in the stem, this patient has sustained a thoracic compression fracture.
  • These are stable injuries that are amenable to a brace as well as ambulation.
  • Bed rest in these patients can be very detrimental due to deconditioning, where are no support of the fracture in the early period can lead to more pain for the patient.
  • This fracture does not require surgical intervention at this time.
57
Q

A newborn is delivered by caesarian section for cephalopelvic disproportion. The exam of the neonate shows an absent anterior fontanelle, broad forehead, and a prominent occiput. The neurologic exam is normal, as is the rest of the exam. Select the correct statement about this patient.

  1. The infant will most likely develop hydrocephalus
  2. Infantile seizures are likely
  3. Neurosurgery consult is needed
  4. There are likely to be cardiac defects
A

3. Neurosurgery consult is needed

  • The findings are consistent with simple, primary craniosynostosis.
  • This involves one suture and is not due to brain growth failure.
  • 10-20 percent of patients have genetic defects.
  • If only one suture is involved there is rarely increased intracranial pressure, but surgery may be considered for cosmetic reasons.
58
Q

A 45-year-old female presents with low backache radiating to the right leg posterolaterally for the past two months. She grades pain as 5 out of 10 on the pain scale. She had renovated her house and shifted some heavy items about two months back, preceding these symptoms. She denies fever, limb weakness, urinary, or bowel disturbances. Physical examination reveals straight leg raise test restricted to 30 degrees. Which of the following medication is the most appropriate for chronic pain relief in this patient?

  1. Gabapentin
  2. Naproxen
  3. Prednisone
  4. Milnacipran
A

1. Gabapentin

  • The patient has lumbosacral radiculopathy. Gabapentin is the most appropriate drug for neurogenic pain relief. It acts by inhibiting voltage-sensitive calcium channels, thereby decreasing neurotransmission of pain signals.
  • NSAIDs can be effective in treating inflammation but has multiple side effects and are not considered to be the first line for neuropathic pain.
  • Adverse effects of long-term steroid use include weight gain, glucose intolerance, irritability, insomnia, and Cushing syndrome.
  • Milnacipran is a serotonin-norepinephrine reuptake inhibitor (SNRI) approved for the treatment of fibromyalgia.
59
Q

Knowledge of the anatomy of the cavernous sinus (CS) is necessary to understand the clinical picture of cavernous sinus syndrome. It is a small but complex structure containing several important structures. Which of the following would be in the CS?

  1. Lacerum segment of the internal carotid artery
  2. Maxillary nerve
  3. Straight sinus
  4. Mandibular nerve
A

2. Maxillary nerve

  • The cavernous sinus is bordered by the temporal bone of the skull and the sphenoid bone, lateral to the sella turcica. The inferior and lateral wall with the roof of CS are extensions of the dura mater.
  • There is a venous plexus in the cavernous sinus. It receives blood from inferior and superior ophthalmic veins and, superficial cortical veins like superior middle cerebral veins.
  • Sympathetic fibers in the cavernous sinus surround the cavernous segment of the internal carotid artery.
  • Cranial nerves (CN) III (oculomotor nerve), IV (trochlear nerve), branches of the V (trigeminal nerve) V1 (ophthalmic) and V2 (maxillary) and VI (abducens nerve) run in the cavernous sinus.
60
Q

A 52-year-old man comes to the outpatient department with the complaint of vague right-sided headache for the past six months. On physical examination, he does not have any neurological deficits. There is a hard swelling of size 5 x 3 cm in the right parietal region. He undergoes a computed tomogram scan, which shows a right temporoparietal hemispheric mass with hyperostosis of the overlying skull, which is eroded by the mass. He undergoes craniotomy and complete excision of the lesion. The pathology report states that the Ki-67 index is 11%. What is the chance of tumor recurrence?

  1. 10%
  2. 20%
  3. 30%
  4. 50%
A

4. 50%

  • The patient has the diagnosis of right temporoparietal meningioma.
  • Meningiomas are known to occur more commonly in elderly individuals. They cause thickening of the bone, in some instances, known as hyperostosis.
  • Ki-67/MIB-1 proliferation index is used to assess the prognosis of meningioma.
  • The recurrence rates for tumors with Ki-67 index 0.7%, 2.1%, and 11% are 9%, 29%, and 50% respectively.
61
Q

A 58-year-old male with no other known neurological condition is receiving a total intravenous anesthetic for a lumbar combined anterior lumbar interbody fusion and posterior spinal fusion under neuromonitoring. Which somatosensory evoked potential response would be most closely be associated with changes in the cortical response from the lumbosacral spine?

  1. N9 is the cortical response from the lumbosacral spine. This response indicates anterior horn function but can not be used to reflect the dorsal horn.
  2. N22 is the cortical response from the lumbosacral spine, which is not an accurate representation of anterior horn function.
  3. P38 is the cortical response from the lumbosacral spine, which is not an accurate representation of the anterior horn of the spinal cord.
  4. N14 is the cortical response from the lumbosacral spine. It represents the function of both the anterior and dorsal horns of the spinal cord, though more accurately reflects the anterior horn.
A

2. N22 is the cortical response from the lumbosacral spine, which is not an accurate representation of anterior horn function.

  • N9 is generated at Erb’s point during somatosensory evoked potentials. SEPs accurately reflect the function of the dorsal column-lemniscal system, whose path runs through the dorsal horn of the spinal cord.
  • N22 is generated from the gray matter of the lumbar spinal cord. SEPs, regardless of recording location, most accurately indicates the function of the dorsal column-lemniscal pathway, which mediates proprioception and mechanoreception. The anterior horn of the spinal cord houses and transmits motor information, which is more accurately reflected with motor evoked potentials or other forms of motor-specific monitoring.
  • P38 is a somatosensory evoked potential generated in the cortex near the representation of the leg. The anterior horn of the spinal cord transmits motor information. SEPs were used in the past as indirect indicators of motor function, but do not truly reflect the motor system. Using SEPs for detecting motor impairment is associated with decreased sensitivity and specificity.
  • N14 is generated in the cervicomedullary region, most likely at the caudal medial lemniscus. All SEPs, including N14, most accurately reflect the function of the dorsal column-lemniscal pathways within the dorsal horn of the spinal cord, not both the anterior and dorsal horns.
62
Q

A 40-year-old woman presents to the clinic with intermittent unilateral headaches. They occur several times a day and last for approximately 30 minutes. They are associated with lacrimation and rhinorrhea. Radiofrequency ablation of the implicated ganglion is planned. Which of the following best describes the innervation of the intended treatment target?

  1. Sympathetics from the cervical ganglion and parasympathetics from the superior salivatory nucleus of the facial nerve
  2. Sympathetics from the superior salivatory nucleus of the facial nerve and parasympathetics from the superior cervical ganglion
  3. Sympathetics from the external carotid plexus and parasympathetics from the superior salivatory nucleus
  4. Sympathetics from the superior salivatory nucleus and parasympathetics from the external carotid plexus
A

1. Sympathetics from the cervical ganglion and parasympathetics from the superior salivatory nucleus of the facial nerve

  • The patient most likely has cluster headaches.
  • Option 1 correctly describes the sphenopalatine ganglion, and this is the best choice since this patient has cluster headaches.
  • Cluster headaches may be treated with radiofrequency ablation of the sphenopalatine ganglion.
  • Option 3 describes the submandibular ganglion.
63
Q

A 65-year-old male with no past medical history presents to the neurosurgery clinic with complaints of chronic neck pain worse with movement with bilateral arm pain and weakness. He failed conservative management. MRI imaging demonstrates diffuse severe central cervical canal stenosis secondary to ligamentum flavum hypertrophy. There are no significant disc herniations. There is a loss of cervical lordosis with a kyphotic deformity. Flexion and extension x-ray shows evidence of instability of the cervical spine. He is deemed to be a surgical candidate. Which of the following would be the most appropriate surgical approach in the treatment of this patient?

  1. Anterior cervical discectomy with fusion
  2. Posterior cervical laminectomy
  3. Posterior cervical laminectomy with lateral mass fusion
  4. Multi-level posterior cervical foraminotomies
A

3. Posterior cervical laminectomy with lateral mass fusion

  • For pathology posterior to the disc space, as in this patient, a posterior approach is warranted. Note that there were no disc herniations present on MRI, making anterior cervical discectomy with fusion (ACDF) a less appealing option.
  • Laminectomy with fusion is indicated given his cervical deformity noted on MRI and evidence of instability on flexion/extension x- rays.
  • Flexion/extension x-rays have good clinical utility when instability is suspected. The diagnosis of instability made decompression with fusion a more suitable option for this patient.
  • Foraminotomies are more indicated in patients with isolated, usually unilateral radiculopathy without significant neck pain.
64
Q

A 65-year-old female with alcohol use disorder is transported by paramedics to the emergency department with severe agitation. On examination, she is confused and irritable. The pulse rate is 110 bpm, blood pressure is 170/100 mm Hg and no fever. She does not have any neurological deficits. She is admitted to the psychiatry ward and treated with thiamine, intravenous fluids, and benzodiazepines. Her serum sodium is found to be 112 mEq/L and is corrected with 3 percent saline over 24 hours, and the repeat sodium value is 136 mEq/L. Her condition deteriorated, and she became non-responsive with agonal breathing. She could not be revived after cardiopulmonary resuscitation. What is the most likely risk factor that led to the unfortunate demise of this patient?

  1. Serum sodium 112 mEq/L for less than 6 hours before admission
  2. Serum sodium 112 mEq/L for 6-12 hours before admission
  3. Serum sodium 112 mEq/L for 12-24 hours before admission
  4. Serum sodium 112 mEq/L for more than 48 hours before admission
A

4. Serum sodium 112 mEq/L for more than 48 hours before admission

  • Central pontine myelinolysis (CPM) is an acute demyelinating condition that primarily affects the pons, although some cases have concomitant extra pontine myelinolysis.
  • It commonly occurs in malnourished patients or patients with alcohol use disorder who undergo rapid correction of chronic hyponatremia.
  • The clinical picture includes seizures, dysphagia, dysarthria, pseudobulbar palsy, behavioral abnormalities, hyperreflexia, quadriplegia, and coma.
  • Serum sodium of less than 120 mEq/L for more than 48 hours is a risk factor for CPM.
65
Q

A 12-year-old male patient is being evaluated for the recent onset of seizures. MRI of the brain revealed a posterior fossa tumor in contact with the four ventricles. After surgical resection, the histopathological evaluation shows that this tumor arose from the cells lining the ventricular system. In pediatric patients, which of the following is the most common location of the tumor?

  1. Intracranial
  2. Spinal cord
  3. Infratentorial
  4. Peripheral nervous system
A

1. Intracranial

  • This patient likely has an ependymoma, which is a tumor arising from the ventricular lining. 90% of ependymomas are intracranial.
  • Ependymomas are most commonly seen in adults 20 to 40 years of age. Intramedullary ependymomas occur most often in the cervical area (60%).
  • The World Health Organization of ependymal tumors includes myxopapillary ependymoma, subependymoma, classic ependymoma, and anaplastic ependymoma.
  • The vast majority of myxopapillary ependymomas occur in the cauda equina (lumbar) area. So this is the most common location in the spine, 40% overall.
66
Q

A 12-year-old boy is brought to the provider because of cerebral palsy since infancy. His mother complains that she has noticed a recent worsening of his body stiffness. Physical examination shows increased tone in upper limbs, with the elbows and wrists flexed. The fingers of the hands are flexed and wrapped around the thumb. The tone in the lower limbs is also more than normal. The resistance to stretch is particularly greater when fast movements are made. The lower limb examination reveals ankle clonus. Further inquiry reveals that the patient had been irregular with his antispasmodic medications. Which among the following is the preferable treatment option for this patient’s condition at this point?
1. Dantrolene sodium
2. Tendon lengthening surgeries
3. Diagnostic nerve block
4. Intrathecal baclofen pump

A

1. Dantrolene sodium

  • Spasticity is a velocity-dependent increase in muscle tone or tonic stretch reflexes associated with hypertonia. A patient may either present with new-onset spasticity or a worsening of pre- existing spasticities, such as in the cases of a long-standing diagnosis of multiple sclerosis or cerebral palsy. Chronic spasticity can worsen after a trigger, which may include skin (ulcers, infections, etc.), visceral (urinary tract infections, constipation, etc.), drug-related (rapid withdrawal of antispasmodic drugs, etc.), or device-related issues (poor seating, ill-fitting orthotic, etc.).
  • Physical examination reveals increased muscle tone in muscle groups, for example, the shoulder abductors, forearm pronators, hip adductor, knee flexors and extensions, plantar flexors, and ankle invertors. A characteristic finding is a ‘thumb in palm’ deformity, with fingers wrapped around the thumb. Additional physical exam findings include clonus, spastic co-contractions, and spastic dystonia.
  • Dantrolene is the preferred agent for spasticities of cerebral origin, such as cerebral palsy or head injury. It blocks the release of calcium from the sarcoplasmic reticulum in the muscles.
  • Diagnostic nerve block and surgery are among the typical choices for treatment of focal spasticity or when the systemic effects of the oral agents are prohibitive at required treatment doses. ITB pump is indicated for patients with generalized spasticity who either cannot tolerate or lack response to more conservative agents (oral, nerve blocks, etc.).
67
Q

A 65-year-old male presents with acute onset of vertigo, dysarthria, dysphagia, and sensory abnormalities involving the right face, the opposite trunk, and extremities. The neurological exam reveals loss of pain and temperature sensation on the same side of the face with decreased pain and temperature sensation on the opposite side of the trunk and extremities. The reduced blood supply in which of the following blood vessels is responsible for this patient’s condition?

  1. Left superior cerebellar artery
  2. Left anterior inferior cerebellar artery
  3. Right superior cerebellar artery
  4. Right posterior inferior cerebellar artery
A

4. Right posterior inferior cerebellar artery

  • Lateral medullary syndrome, or Wallenberg syndrome, is caused by occlusion of one of the posterior inferior cerebellar arteries. This results in infarction of the lateral medulla. The other artery occlusion causing the syndrome is one of the vertebral arteries.
  • Patients have sensory deficits affecting the ipsilateral face, contralateral extremities, and trunk.
  • The specific sensory loss includes pain and temperature sensation.
  • Clinical symptoms include dysphagia, ataxia, facial pain, slurred speech, vertigo, diplopia, Horner syndrome, nystagmus, and palatal myoclonus.
68
Q

A 40-year-old woman, with a serious head injury, was brought to the emergency department following a motor vehicle collision. Her CT scan showed a skull fracture involving the foramen rotundum. Which of the following nerves is most likely affected in this case?
1. Sural Nerve
2. Hypoglossal Nerve
3. Maxillary Nerve
4. Vagus Nerve

A

3. Maxillary Nerve

  • Maxillary nerve is the second division of the trigeminal nerve and arises from the trigeminal ganglion.
  • Maxillary nerve leaves the middle cranial fossa through foramen rotundum to enter the superior part of the pterygopalatine fossa.
  • Contents of foramen rotundum include maxillary division of the trigeminal nerve, artery of foramen rotundum, and emissary veins.
  • Foramen rotundum is a circular hole within the sphenoid bone connecting the middle cranial fossa with the pterygopalatine fossa.
69
Q

A 48-year-old left-handed man with a past medical history of hypertension, hyperlipidemia, uncontrolled diabetes, and newly diagnosed untreated paroxysmal atrial fibrillation presents to the emergency department with the sudden onset of nausea, dizziness, dysphagia, ataxia, and an ipsilateral sensory impairment over the face, as well as Horner syndrome. The episode started five hours ago. Vital signs show a temperature of 38 C, pulse rate of 120/min, blood pressure of 184/109, mmHg, and respiratory rate of 20/min. The patient is very confused, has visible ptosis in his right eye, has ataxia, and is complaining of severe nausea and dizziness. His blood glucose level is 110 mg/dL. CT head without contrast is negative for acute bleeding, and subsequent CT angiogram of the head and neck shows a large vessel occlusion (LVO) in the proximal basilar artery and lack of flow in the distal right vertebral artery. Which of the following is the next best step in managing this patient?

  1. Alteplase administration
  2. CT perfusion scan to measure the extent of the infarct
  3. Mechanical thrombectomy
  4. Endovascular deconstruction of vertebral artery
A

3. Mechanical thrombectomy

  • Mechanical thrombectomy (MT) is recommended in this case based on the recent DAWN and DEFUSE-3 trials for anterior circulation.
  • Unfortunately, this patient is outside the alteplase window, and tPA is not indicated in patients with acute ischemic strokes presenting 3 to4.5 hours after the onset of the symptoms. Alteplase administration, as per NINDS trial results published in 1995, showed significant mortality and outcomes benefits of alteplase administration if the patient has arrived in the emergency department within the three hours window and has no contraindication to alteplase. Later, studies showed similar benefits within 4.5 hours of the last known well, but after that, there was no recommendation for alteplase administration.
  • The patient is eligible for mechanical thrombectomy (MT) for an LVO in the basilar artery. Later, he should be assessed for vertebral artery compression as the etiology of his infarct. There is no need to delay the treatment with unnecessary testing and alteplase, and the patient should immediately receive the MT.
  • External compression of the vertebral artery (Bow Hunter syndrome or vertebrobasilar insufficiency) can cause posterior circulation stroke, which needs either endovascular deconstruction or reconstruction, external surgical decompression, or surgical bypass techniques, which is likely the case of this patient, but at this time, he should be moved to an interventional radiology suite for the MT and later should be assessed for vertebral artery decompression surgery.
70
Q

A 65-year-old woman with a history of schwannoma presents for her pre-operative examination. MRI reveals that the schwannoma occupies the left cerebellopontine angle (CPA). Her initial presenting symptoms were progressive hearing loss, tinnitus, and vertigo. Vital signs reveal blood pressure of 130/85 mmHg, heart rate of 80 beats per minute, respiratory rate of 15 breaths per minute, and temperature of 98.9 F. What would most likely be found on this patient’s physical examination?

  1. Left facial droop
  2. Right lateralization on the Weber test
  3. Left lateralization on the Weber test
  4. Bone conduction is greater than air conduction bilaterally on the Rinne test
A

2. Right lateralization on the Weber test

  • Vestibular schwannomas (acoustic neuromas) are tumors of Schwann cell origin that affect the vestibulocochlear nerve, and they are the most common cerebellopontine angle (CPA) tumors. Bilateral vestibular schwannomas are the hallmark sign of Neurofibromatosis Type 2 (NF-2), an autosomal dominant genetic syndrome.
  • The usual presentation of vestibular schwannomas (acoustic neuromas) is progressive, unilateral sensorineural hearing loss, and commonly associated symptoms are tinnitus and vertigo. Using the Weber and Rinne tests, sensorineural hearing loss can be distinguished from conductive hearing loss. With sensorineural hearing loss, the Rinne test would show that air conduction is greater than bone conduction bilaterally, and the Weber test would lateralize to the unaffected ear.
  • Treatment options for vestibular schwannomas (acoustic neuromas) are microsurgical resection, radiation therapy, and observation. MRI is the diagnostic imaging of choice. Early diagnosis and treatment are important to decrease the likelihood of these complications, such as CSF leak and meningitis.
  • Facial nerve (CN VII) palsy (facial droop) rarely occurs with vestibular schwannomas (acoustic neuromas) because the facial nerve is resistant to chronic pressure.
71
Q

A 66-year-old woman presents to the clinic with severe back pain that started after a ground-level fall two days ago. Since last night she complains of tingling in her lower extremities and states the pain is radiating to the inguinal area. On examination, she has mild perianal numbness. X-ray reveals the collapse of the L1 vertebra with an anterior vertebral height reduced by 4 mm. Lumbar computed tomography scan shows retropulsed bone in the canal with 40% stenosis. What is the most appropriate management strategy for this patient?

  1. Vertebroplasty
  2. Decompressive laminectomy, instrumentation, and fusion
  3. External thoracolumbar orthosis
  4. Kyphoplasty
A

2. Decompressive laminectomy, instrumentation, and fusion

  • Acute traumatic vertebral fractures with significant compression of the spinal cord should be managed surgically with decompression.
  • Vertebroplasty and kyphoplasty provide structural support to a fractured vertebra but do not remove bone fragments inside the spinal canal.
  • Indications for vertebroplasty and kyphoplasty include vertebral compression fracture, which causes significant physical limitations, including back pain, functional disability, and increasing kyphosis after a period of 4-6 weeks of conservative therapies.
  • Relative contraindications for vertebroplasty and kyphoplasty include fracture extension into the posterior vertebral body wall that can risk cement extravasation into the spinal canal, and severe compression fractures/deformity.
72
Q

A 60-year-old female patient known with diabetes and hypertension presented with left-sided weakness, intact sensory exam, and dysarthria. There is a history of sudden dizziness and loss of balance of the patient. Her vital signs are blood pressure: 145/90 mmHg, pulse rate: 65 beats per minute, and respiratory rate: 17 breath per minute. Her CT head was ordered immediately, which showed no acute bleed. She was started on thrombolytic therapy. Twenty-four hours later, the patient had multiple episodes of vomiting and complained of a headache. Upon repeat head CT, cerebral edema was noted. What was the most likely underlying cause of these CT scan changes?

  1. Formation of peroxide and nitro oxide-free radicles causing endothelial damage
  2. Head trauma that resulted in the development of cerebral edema
  3. Decreased oxygen supply causing damage of the endothelial lining
  4. Increased blood pressure leading to increased cerebral blood flow causing edema
A

1. Formation of peroxide and nitro oxide-free radicles causing endothelial damage

  • The patient’s presentation of one-sided weakness and dysarthria is indicative of cerebral ischemia leading to an infarct. Over time the thrombus breaks down, and thrombolytic therapy with antiplatelet drugs and aspirin helps increase the blood flow and oxygen supply to the tissue.
  • During the ischemic period, adenosine triphosphate (ATP) depletion occurs, and a shift to anaerobic respiration. This results in damage to the membrane as well as mitochondria. When revascularization occurs, and there is increased oxygen and blood flow, the formation of reactive species takes place. Besides, neutrophils flow in, and activation of the inflammatory process occurs.
  • Due to this damage, the capillary permeability is markedly increased, and the increased blood flow causes increased pressure resulting in cerebral edema.
  • There is no history suggestive of trauma, and a sudden increase in blood pressure can result in hemorrhage and not in cerebral edema.
73
Q

A 70-year-old man is brought to the emergency department following a history of fall from his bicycle while trying to save a child who was trying to cross the road. He fell down with his face hitting the ground. On examination, he is fully conscious and oriented. He has a small subgaleal hematoma in the frontal region. He does not have any motor deficits in his limbs or numbness in the extremities. What is the next step in management?
1. Computed tomography of the head and spine
2. Computed tomogram scan of the cervical spine
3. Magnetic resonance imaging of the cervical spine
4. Observation for 24 hours

A

1. Computed tomography of the head and spine

  • The patient is at high risk of cervical spine injury as per the Canadian cervical spine rule due to his age, which is more than 65 years.
  • According to the Canadian cervical spine rule, radiographs should be obtained in those patients with high-risk factors, including age older than 65 years, dangerous mechanism of injury, and paresthesia in extremities mandating radiography.
  • As the patient is above, 65 CT of the brain is needed. CT of the cervical spine should be done as well.
  • Consider magnetic resonance imaging if neurologic signs or symptoms are present, and plain radiographs and/or CT scans are normal
74
Q

A 65-year-old female is brought to the emergency department with the complaint of sudden onset unresponsiveness for the past two hours. She has a history of hypertension and diabetes mellitus type 2. On examination, her blood pressure is 180/110 mm Hg, and her pulse rate is 50 per minute. Her Glasgow Coma Scale score is E2V2M5, and her pupils are bilaterally 4 mm in size, reacting to light. A computed tomogram of the brain is ordered, which shows hemorrhage filling the suprasellar cistern. Vascular abnormality in which of the following blood vessels can be the most likely cause?

  1. Anterior cerebral artery
  2. Middle cerebral artery bifurcation
  3. Anterior communicating artery
  4. Posterior communicating artery
A

4. Posterior communicating artery

  • The patient has subarachnoid hemorrhage filling the suprasellar cistern, possibly from an aneurysm.
  • Posterior communicating artery aneurysms on rupture tend to bleed into the adjacent suprasellar cistern.
  • Sylvian fissure subarachnoid hemorrhage should suggest a middle cerebral artery bifurcation aneurysm.
  • Anterior communicating artery aneurysm rupture tends to collect as a subarachnoid hemorrhage in an interhemispheric location.
75
Q

A 66-year-old woman with a history of melanoma diagnosed 5 months ago, presents to the clinic with a complaint of severe neck pain radiating to her left arm. The pain is 7/10 in intensity, which started 4 days ago and is not relieved with NSAIDs. She also complains of paresthesias in the left arm. Her husband complains that lately, he has seen her talking to herself a lot. He also reports that her sleep is disturbed, and she had one episode of loss of consciousness for few seconds 2 days ago. Otherwise, she performs individual and household work herself. On examination, she is conscious and oriented. Cranial nerves examination is normal. Left-arm has a sensory loss to sharp pain at C5 and C6 levels. No muscle weakness is noted. Reflexes are normal. The rest of the neurological exam is unremarkable. MRI brain and neck with contrast shows cranial nerve root and nodular leptomeningeal enhancement at C4-C6 levels. Lumbar puncture cytology shows malignant cells. Which of the following interventions is most likely to improve the patient’s pain?

  1. Cervical dexamethasone injection
  2. Epidural pain injection
  3. Focal radiotherapy
  4. Vemurafenib
A

3. Focal radiotherapy

  • Melanoma is one of the most common tumors that can cause metastasis in the leptomeningeal layers of the CNS. The dissemination of malignant cells from the primary tumor to leptomeninges can occur at multiple levels, which is responsible for causing a wide array of presenting symptoms in carcinomatous meningitis (CM). Spinal cord involvement in CM can cause hypoesthesia and paresis depending upon whether sensory or motor root is affected. Spinal cord involvement can cause spinal radiculopathy.
  • MRI has got diagnostic significance. MRI of the spinal cord shows linear and nodular leptomeningeal enhancements along the nerve roots, particularly along the cauda equina. MRI has another role that enhances the path of malignant cell seeding, which can be helpful for radiotherapy treatment.
  • Management of a patient with carcinomatous meningitis is complex. It includes a combination of both systemic and intrathecal chemotherapy with or without radiotherapy. This combination improves clinical outcomes. Radiotherapy is more commonly used for symptomatic and palliative treatment rather than as a means for eradicating the disease. Focal radiotherapy is used in the case of nodular deposits of malignant cells in meninges. Melanomas are not uniformly radioresistant, with a significant subset displaying inherent radiosensitivity.
  • Cranial irradiation is helpful in symptomatic CM by reducing focal tumor growths involving cranial nerves. The neurological deficit may not improve, but radicular pain does. The dosage used for cranial and spinal irradiation is 30 Gy in 10 daily fractions. Vemurafenib is a BRAF inhibitor administered systemically and shows a significant clinical response in patients with lepto-meningeal metastasis in melanoma patients. However, the management of symptoms like pain caused by nodular deposits of leptomeningeal metastasis is best treated with focal radiotherapy.
76
Q

A 19-year-old college football lineman presents to the clinic with axial back pain that began several weeks ago abruptly during practice while participating in blocking drills. The pain is primarily on the left side at L4-L5. An x-ray performed in the office demonstrates a lucency between the superior and inferior articular processes on the left side at L5. What is the only potentially pathognomonic physical exam finding associated with this injury and how is it performed?

  1. Straight leg raise; patient lying supine with legs straight, and one leg is passively raised. The reproduction of radicular pain indicates a positive test.
  2. Lumbar facet loading; patient seated and placed in extension and rotation toward the affected side while an axial load is placed through the shoulders towards the affected side. The reproduction of low back pain indicates a positive test.
  3. Step off deformity; the patient is seated in a neutral position and using palpation technique examiners hand glides along spinous processes, beginning at the sacrum and moving cephalad. Significant “drop off” from one level to the next indicates a positive test.
  4. Stork test; the patient stands on one leg and is placed in hyperextension. Reproduction of pain while standing on the ipsilateral leg associated with the side of low back pain indicates a positive test.
A

4. Stork test; the patient stands on one leg and is placed in hyperextension. Reproduction of pain while standing on the ipsilateral leg associated with the side of low back pain indicates a positive test.

  • A lucency between the superior and inferior articular processes on x-ray suggests a fracture through the pars interarticularis (also known as spondylolysis).
  • Spondylolysis is a common cause of back pain in adolescent athletes, especially those in sports requiring repetitive lumbar hyperextension.
  • The stork test is also called a single leg hyperextension test.
  • Even with a positive stork test, diagnosis is confirmed with imaging demonstrating a fracture through the pars interarticularis.
77
Q

A 35-year-old patient presents with a slowly progressive headache for the last three months. His MRI brain images showed features highly suggestive of an insular lobe low- grade glioma. What is the currently recommended management plan for the patient?

  1. Regular follow up
  2. Radiation
  3. Gross total resection of the tumor
  4. Maximal safe resection of the tumor
A

4. Maximal safe resection of the tumor

  • The management of the insular region low-grade glioma is still a debatable issue, mostly because of the eloquence of the region involved.
  • There is a high rate of transition of low-grade glioma into a high- grade variant. So, it is recommended for early histological confirmation of the tumor grade, which will help in determining further adjuvant chemo-radiation.
  • Maximal safe resection is now the mainstay surgical goal while operating in tumors in eloquent areas.
  • Gross resection increases the risk of neurological deficits in the patient. The transition of the tumor into high grade during the follow-up period can adversely affect the prognosis, periods of recurrence-free survival as well as overall survival in the patient.
78
Q

A 65-year-old male patient received epidural anesthesia for total knee arthroplasty with no complications during the procedure. The next day he started complaining of bilateral leg weakness that progressed to paralysis with no associated pain. What is the cause of the most likely diagnosis in this patient?

  1. Accidental dural puncture with an epidural needle
  2. Nerve injury
  3. Epidural space infection
  4. Blood accumulation and ischemic compression of the spinal cord
A

4. Blood accumulation and ischemic compression of the spinal cord

  • Accidental puncture of the dura mater with the epidural needle or wet tap presents with a headache. Bilateral leg weakness is more consistent with the physical compression of the spinal cord that can be explained by hematoma formation.
  • Nerve root injury can happen when providing epidural anesthesia. However, the presentation is usually unilateral and symptoms manifest in the muscular and dermatomal region that corresponds to the injured nerve root. The bilateral complete paralysis that the patient presents in this case, can be better explained by a lumbar epidural hematoma.
  • The epidural abscess formation typically takes more than 24 hours and is accompanied by signs of infection. An epidural hematoma can present accurately as evidenced in this patient.
  • An epidural hematoma can be formed by bleeding inside the epidural space. Ischemic compression of the spinal cord can cause permanent neurologic damage. Epidural hematoma is the most likely explanation for this patient considering the acute presentation, paralysis and lack of accompanying infection signs.
79
Q

A 17-year-old male presents with complaints of a recent decrease in his peripheral vision. He developed polyuria and polydipsia six months ago and is currently receiving treatment for diabetes insipidus. On neuro-ophthalmologic examination, the patient is alert and oriented with intact extraocular movement. However, there is bitemporal hemianopsia and decreased visual acuity. Computed tomography (CT) scan of the head with and without contrast revealed a sellar tumor with suprasellar extension. Histological examination demonstrated a biphasic cellular population composed of large malignant epithelioid cells and small lymphocytes. Laboratory evaluation of tumor biomarkers showed an alpha-fetoprotein level of 1 ng/L and a beta-human chorionic gonadotropin (hCG) of 0.001 IU/L. Which of the following is the most important factor in the staging of the tumor?

  1. Evaluation of surgical resection margins
  2. Genetic analysis
  3. Molecular profiling
  4. Neuraxis magnetic resonance imaging (MRI) and cerebrospinal fluid (CSF) cytology
A

4. Neuraxis magnetic resonance imaging (MRI) and cerebrospinal fluid (CSF) cytology

  • It is critical to determine the extent of disease before initiating treatment because the approach is different for localized versus disseminated germinoma. Neuroimaging with MRI and lumbar puncture for CSF cytology is important for tumor staging.
  • The presence of tumor cells on CSF cytology is an indication of disseminated disease (M+), and patients are treated more aggressively with craniospinal irradiation for better outcomes.
  • There are no recommended TNM-based staging guidelines for intracranial tumors. However, the modified Chang system used for medulloblastoma can be employed in germinoma.
  • Seminomas are highly radiosensitive and chemosensitive with an excellent prognosis. The role of surgical resection in germinoma is not well established.
80
Q

A 66-year-old male farmer, who needs to carry heavyweight during his work, presents to the clinic with chronic low back pain without radiating to lower legs. For differentiating purposes, a facet block will be applied. Regarding the facet block, which of the guiding method is the most cost-effective?

  1. Computed tomography (CT) guided
  2. Magnetic resoannce imaging (MRI) guided
  3. Ultrasound-guided
  4. Landmark guided
A

3. Ultrasound-guided

  • Each facet joint receives innervation from two spinal levels, one of which is from the dorsal ramus at the same vertebral level and the other of which is from one level above.
  • Therefore, two levels of the medial branch should be blocked.
  • An ultrasound-guided facet block is easy to perform in a clinic setting and without radiation exposure.
  • Facet-related pain is not easy to locate due to dual innervation by two medial branch of the dorsal ramus.
81
Q

A 49-year-old man is admitted to the ICU with the placement of an external ventricular drain (EVD) for grade 4 subarachnoid hemorrhage (Hunt and Hess scale) and hydrocephalus. A day after the EVD placement, frank blood is noticed to be draining from the EVD. The patient is comatose with a GCS score of 3. Vital signs show a blood pressure of 180/110 mmHg. Which of the following is the most likely cause of the patient’s current presentation?
1. Rebleed
2. Vasospasm
3. Brain herniation
4. Cerebral infarction

A

1. Rebleed

  • This scenario describes a case of rebleed due to aneurysm re- rupture.
  • Rebleeding is one of the most dangerous complications of subarachnoid hemorrhage, with a majority occurring within the 1st 6 hours and usually within the 1st 2 weeks.
  • The most effective treatment to prevent rebleed is through early surgical treatment of the aneurysm.
  • Risk factors for rebleed include hypertension, larger aneurysm size, and a high Hunt and Hess scale.
82
Q

A 17-year-old female presents with 2 stroke-like episodes in the past year. She suffers from migraine headaches for the past 8 years and has been admitted several times for generalized weakness, loss of appetite, and vomiting. Previous admissions reveal elevated lactic acid levels. A muscle biopsy done shows ragged red fibers. Which of the following treatment options are appropriate for this patient?

  1. Intravenous immunoglobulin
  2. Plasma exchange
  3. Corticosteroids
  4. Coenzyme Q10
A

4. Coenzyme Q10

  • This patient is suffering from mitochondrial encephalomyopathy with lactic acidosis and stroke-like episodes (MELAS).
  • Although no specific treatment for mitochondrial encephalomyopathy with lactic acidosis and stroke-like episodes (MELAS) exists, respiratory chain cofactors are often recommended, including coenzyme Q10.
  • Avoidance and early treatment of any disease that may increase the metabolic rate, such as infections, should be done.
  • Rehabilitation and long-term planning after encephalopathy stroke-like episodes are also recommended.
83
Q

A patient was brought for consult due to intermittent involuntary movements. The movements would start from the hand and gradually involve the whole arm, face, and leg on one side. This would progress in a matter of seconds, would last for a minute, and completely resolve. There is no loss of consciousness after the episode. These episodes happen a couple of times a day for the past two weeks. On examination, there are no cranial nerve deficits, and the motor and the sensory exam were normal. Babinski was negative bilaterally. Which one of the following is most likely in this patient?

  1. Jacksonian seizure
  2. Juvenile myoclonic seizure
  3. Absence seizure
  4. Focal impaired awareness seizure
A

1. Jacksonian seizure

  • These seizures are of brief duration and can occur many times a day. They are more common in children and afterwards there is no confusion, but memory deficits can exist.
  • Juvenile myoclonic seizures show jerking in the neck, shoulder and upper arm usually occurring in the morning.
  • Petit mal epilepsy shows abrupt onset and impairment of consciousness, interruption of ongoing activity, and some jerking of the face and hand. Classic spike and wave form on EEG.
  • Partial complex seizures show repetition of inappropriate acts, automatisms such as lip smacking, grimacing and chewing, forced turning of eyes followed by an indefinite period of confusion and garbled speech. This is also called psychomotor or temporal lobe epilepsy. There is confusion for a short period after the seizure, amnesia regarding events, difficulty talking, and loss of judgment.
84
Q

A 60-year-old woman presents to the clinic with a painful neck for one month. There is no history of trauma or fever. On physical examination, there is a painful neck range of motion. The small joints of her hands are swollen with ulnar deviation at the metacarpophalangeal joints. The MRI reveals atlantoaxial subluxation with atlantodental interval (ADI) of 5 mm. What is the most appropriate management strategy for this patient?

  1. C1-C2 fusion
  2. Conservative measurement
  3. Occipito-cervical fusion
  4. C2-C3 fusion
A

2. Conservative measurement

  • The patient mentioned above has rheumatoid arthritis associated with cervical instability.
  • Flexion and extension radiographs are required for definitive measurement of cervical subluxation/instability.
  • In normal people, ADI > 3.5 mm is significant, but In rheumatoid arthritis, the limit of conservative measurement is 10 mm ADI.
  • Cervical fusion is required after ADI crosses the 10 mm limit.
85
Q

During a pelvic MRI for ovarian cysts, a spinal cyst is found to be surrounding a traveling nerve root. There are no fibers in the wall itself, and the sac is continuous with the dura. There is no compression of the cauda equina itself. What is the next best step in management?

  1. Reassure
  2. Refer to spinal service for surgery
  3. Physiotherapy
  4. Whole spine MRI
A

1. Reassure

  • Tarlov cyst is the described finding here on the MRI, and no specific treatment is required if asymptomatic.
  • Surgery would not be considered for an asymptomatic Tarlov cyst.
  • Physiotherapy may be considered as a conservative treatment if symptomatic but not clearly indicated in this case.
  • A whole spine MRI will provide little diagnostic value as the diagnosis itself is fairly clear.
86
Q

A 60-year-old man presents after a seizure. The patient has had a fever and headaches for the past week, for which he was given paracetamol. Vital signs show oxygen saturation of 98% on room air, respiratory rate of 18 per minute, heart rate of 54 beats per minute and regular, blood pressure of 140/70 mmHg, and temperature of 39.2 C (102.5 F). On examination, the patient is drowsy, does not follow commands, and withdraws the right more than the left extremity to pain. His right pupil is slightly dilated when compared to the left, and it is sluggishly reacting to light. Babinski’s sign is positive bilaterally. He is positive for nuchal rigidity and Brudzinski’s sign. A head computed tomography (CT) scan reveals edema of both cortical hemispheres - right more than left. The herniation of what part of his brain can explain his eye findings?

  1. Subfalcine herniation of the cingulate gyrus
  2. Transtentorial herniation of the medial temporal lobe
  3. Cerebellar tonsillar herniation
  4. Transtentorial herniation of the cerebellum
A

2. Transtentorial herniation of the medial temporal lobe

  • Uncal herniation is the most common subtype of transtentorial herniation. The herniation of the most medial part of the temporal lobe (uncus) across the tentorial incisura is called uncal herniation.
  • The uncus squeezes the third cranial nerve, and this causes dilation and failure of constriction of the pupil on the affected side.
  • Uncal herniation is usually caused by a supratentorial mass lesion causing an increase in the intracranial pressure. Here the mass can compress the brainstem and the posterior cerebral arteries.
  • Uncal herniation may be unilateral or bilateral.
87
Q

A 15-year-old boy presents to the emergency department after a drowning. He is intubated and on mechanical ventilation on arrival. He is unresponsive. Vital signs reveal hypotension and hypothermia with tachycardia. On neurological examination, his brainstem reflexes are absent. The apnea test does not show any spontaneous respiratory drive. What is the next best step in the management of this patient?

  1. Declare brainstem death
  2. Perform cerebral angiography
  3. Repeat brainstem reflexes and apnea test after 6 hours
  4. Repeat brainstem reflexes and apnea test after 12 hours
A

4. Repeat brainstem reflexes and apnea test after 12 hours

  • In patients who present with the absence of brainstem reflexes after a drowning, hypothermia can be one of the confounding factors.
  • The most rational approach is to perform and validate both the brainstem reflexes and the apnea test through repeat examination after 12 hours.
  • it is prudent to watch for pitfalls and confounding factors such as drugs, intoxication, hypotension, hypothermia, etc while evaluating brain stem functions.
  • Cerebral angiography is not required if both the brainstem reflexes and the apnea test criteria are correctly fulfilled.
88
Q

A 55-year-old man presents to the clinic with a long-standing history of low back pain and stiffness. Plain radiographs of his cervical and lumbar spine show flowing wax-like ossification along the anterior margins of his cervical vertebral bodies with preserved disc spaces. Which of the following complications is most often associated with the patient’s diagnosis?

  1. Difficult airway/ intubation for anesthesia
  2. Osteopenia
  3. Loss of cervical intervertebral disc height
  4. Bone formation between vertebral bodies
A

1. Difficult airway/ intubation for anesthesia

  • The clinical presentation of diffuse idiopathic skeletal hyperostosis (DISH) is often an older patient with moderate back pain and stiffness. Soft tissue involvement secondary to osteophytes at the cervical segments in DISH may result in dysphagia, hoarseness, sleep apnea, and difficult intubation. Further evaluation with a swallow study or consultation to otolaryngology or gastroenterology may be warranted.
  • Typically, the clinical symptoms are mild in comparison to the extensive radiographic findings that are used to diagnose DISH. As such, DISH is often discovered incidentally in asymptomatic patients.
  • The diagnostic criteria for DISH is composed of three findings: 1) anterolateral vertebral flowing ossification at a minimum of four contiguous levels; 2) preservation of disc height and lack of significant degenerative changes at the involved segments, which differentiates from degenerative spondylosis; and 3) absence of ankylosis at the facet-joint interface and absence of sacroiliac erosion, osteopenia, sclerosis, or osseous fusion, which differentiates from ankylosing spondylitis.
  • The differential diagnoses for patients with back pain, stiffness, and spondylophytosis also include ankylosing spondylitis (AS), seronegative spondyloarthropathies, Charcot spine, acromegaly, psoriasis, Reiter’s syndrome, pseudogout, and hypoparathyroidism Disc space ossification, marginal syndesmophytes, bilateral sacroiliac involvement, squaring of vertebral bodies are all associated with ankylosing spondylitis (AS).
89
Q

A 2-year-old boy undergoes surgical resection of neuroblastoma. The description of the patient’s surgical staging is the following: a localized tumor with complete gross excision, with ipsilateral lymph nodes positive for tumor cells, and enlarged contralateral lymph nodes that are negative for tumor cells. What is the surgical stage of the patient’s condition according to the International Neuroblastoma Staging System (INSS)?

  1. Stage 1
  2. Stage 2A
  3. Stage 2B
  4. Stage 3
A

3. Stage 2B

  • Neuroblastoma (NB) is the most frequently occurring extracranial childhood tumor. It is classified as an embryonal neuroendocrine tumor, originating from neural crest progenitor cells.
  • The International Neuroblastoma Staging System (INSS) based on surgical resection is used to stage neuroblastomas.
  • According to the INSS, Stage 2B consists of a localized tumor with complete gross excision, with ipsilateral lymph nodes positive for tumor cells and enlarged contralateral lymph nodes negative for tumor cells.
  • Given the heterogeneity in tumor location, grade, and stage at diagnosis, treatment modalities include simple observation, surgical resection, chemotherapy, radiation therapy, stem cell transplantation, and immunotherapy.
90
Q

A 35-year-old male presents to the clinic with sudden onset back and leg pain after starting training as a powerlifter. The pain started after lifting a heavy bag that weighed over 200 lbs (91 kg). He heard a loud pop after his first repetition and immediately fell to the ground in pain. He endorses moderate pain in his right lower lumbar region that radiates down his leg to below the knee. He denies any weakness or sensation changes and has been able to ambulate without difficulty. The pain is manageable with ibuprofen. The straight leg raise test reproduces his pain, but his physical exam is otherwise unremarkable. The patient’s accident occurred five days ago. How long, on average, should it take for him to get back to the previous level of training and weight lifting?

  1. Twelve months
  2. Six months
  3. Three months
  4. Six weeks
A

2. Six months

  • The herniated portion of the disc regresses over time, with partial to complete resolution after six months in two-thirds of people. Thus six months would be the minimal recommendation until the disc regresses enough to bear the stress, such as lifting heavyweights.
  • In most cases, the pain will get better and completely resolve in 4 to 6 weeks.
  • A separate study showed at three months following injury, approximately 30 percent of patients still had complaints of some back pain.
  • About 10% of people have sufficient severe pain after six weeks for surgery to become a consideration following a herniated disc.
91
Q

A 10-year-old boy presents with nausea, vomiting, headache, and lethargy for two days. He had a ventriculoperitoneal shunt placed one year ago for congenital obstructive hydrocephalus. His vital signs are within normal limits. Examination shows a lethargic boy with a soft and nontender abdomen and normal neurologic functioning. What is the best initial imaging study for this patient’s condition?

  1. Shunt series radiographs
  2. Contrast-enhanced head CT
  3. Phase-contrast MRI
  4. Radionuclide shunt study
A

1. Shunt series radiographs

  • Congenital obstructive hydrocephalus is associated with a high risk of developing shunt complications, most commonly shunt obstruction that characteristically presents with headache, nausea, vomiting, and lethargy.
  • Shunt series radiographs of the shunt components obtained in at least two planes is the imaging method of choice in the initial work-up of patients with suspected ventriculoperitoneal shunt obstruction. Low-dose, non-contrast CT is an imaging option used as an alternative to the shunt series for evaluating shunt function. However, contrast-enhanced CT is not used for this purpose.
  • Phase-contrast MRI is an imaging method used for the qualitative and quantitative evaluation of cerebrospinal fluid flow. It is not used in the evaluation of suspected ventriculoperitoneal shunt failure.
  • Radionuclide shunt study is a technique that can be used in the evaluation of the ventriculoperitoneal shunt malfunction, but not as the first-line modality.
92
Q

A 58-year-old woman is seen on postoperative day 1 following acoustic neuroma removal. She is found to have new-onset unilateral facial droop. Which of the following nerve lies adjacent to the nerve that was most likely injured in the bony part of posterior cranial fossa?

  1. vagus nerve
  2. vestibulocochlear nerve
  3. spinal accessory nerve
  4. Abducens Nerve
A

2. vestibulocochlear nerve

  • The facial nerve is most likely injured given her facial weakness. The facial nerve and vestibulocochlear nerve run together within the internal auditory canal, found within the posterior fossa. The internal auditory meatus is a canal in the petrous part of the temporal bone that contains the facial and vestibulocochlear nerves. The vagus nerve exits via the medulla and exits the skull through the jugular foramen
  • The internal auditory meatus is a canal in the petrous part of the temporal bone that contains the facial and vestibulocochlear nerves. The vagus nerve exits via the medulla and exits the skull through the jugular foramen.
  • The spinal accessory nerve does not enter the internal auditory canal. It runs through the jugular foramen along with CN IX and X.
  • While the abducens nerve exits the pons medial to the facial nerve, it does not enter the internal auditory canal and exits the skull via the superior orbital fissure.
93
Q

A 74-year-old man presents to the clinic with a small scar superior to his right eyebrow. He has had recent frostbite, as well as an active infection at the site. Cryoablation is being considered. Which of the following possible conditions at the lesion is the safest to proceed with the procedure?

  1. Very thin skin
  2. Proximity to desired hair (eyebrow etc.)
  3. Infection
  4. Contracture/ scar tissue
A

4. Contracture/ scar tissue

  • Skin damage can be seen, especially if the probe is too superficial. Further locations on the face or highly visible locations as there is a risk of hyperpigmentation or hypopigmentation at the ablation site.
  • Alopecia may also occur at the cryoablation site, which can be a concern for the eyebrow during supraorbital lesions.
  • Infection is a possible adverse effect any time a needle punctures the skin. If an infection is already present, the procedure should be performed after the resolution of symptoms.
  • There have no reported cases of contracture as a result of cryoablation. Cryoablation has been used to treat contractures, including Dupuytren contracture as well as cosmetic scar tissue reduction.
94
Q

A 77-year-old male presents to the emergency department complaining of neck pain. Physical exam demonstrates an inability to passively move his neck secondary to pain, tenderness to palpation in the neck posteriorly. Neurologic exam demonstrates symmetric bilateral strength in upper and lower extremities with no deficits, however, he is complaining of numbness and tingling in his small finger bilaterally. Which of the following is the next best step in the management of this patient?

  1. Immediate surgical decompression
  2. Halo-Vest application followed by urgent MRI
  3. Cervical orthosis followed by imaging of the cervical spine
  4. Nerve conduction studies to evaluate for ulnar nerve injury
A

3. Cervical orthosis followed by imaging of the cervical spine

  • Greater than 60% of cervical spine injuries in the geriatric population are secondary to falls. An elderly patient is presenting to the emergency department with neck pain and bilateral upper extremity sensory changes. Proper management would be to immobilize the cervical spine to prevent further injury, and imaging of the spine to determine the extent of the injury.
  • One-third of geriatric patients in the community fall each year.
  • Geriatric patients can suffer cervical spine injury with minor mechanisms, including ground-level falls.
  • Motor vehicle accidents are the second most common cause of cervical spine injury in geriatric patients. Geriatric patients are more likely to be injured in minor motor vehicle accidents, and have a higher rate of motor vehicle accidents compared to their younger cohorts.
95
Q

A 50-year-old female comes in complaining of sudden onset pain in bilateral lower extremities and loss of bladder function. Her physical examination reveals a motor weakness in her left lower extremity 3/5 compared to the right along with diminished sensation to light touch, pinprick, and temperature along L5 and Si dermatomes on the right compared to the left. The rest of her physical, musculoskeletal, and neurologic examination is normal. Lumbosacral x-rays done by her primary care physician demonstrate anterolisthesis of L5 on S1. Which of the following is the most appropriate immediate action?

  1. Consult the spine surgeon
  2. Intravenous Opioids
  3. Physical therapy
  4. Reassurance and return to home with a follow-up visit in 2 weeks if symptoms persist
A

1. Consult the spine surgeon

  • Considering the acute onset of bladder dysfunction and neurologic deficits on physical examination along with the anterolisthesis of L5 on S1, urgent evaluation by a spine surgeon seems to be the best immediate option of all.
  • This patient needs further workup and possibly even urgent intervention by the spine surgeon at this time.While intravenous opioids can be used to control acute pain, they by no means should be considered adequate in managing this situation that demands urgent surgical attention.
  • Physical therapy may be considered in future for this patient for physical rehabilitation once surgical evaluation and /or intervention has been completed. Physical therapy for acute pain management is inappropriate for this case considering the risk of neurologic deficits that may ensue from further movement of an unstable spine.
  • This condition could be a surgical emergency and so this patient should be actively managed in an in-patient setting. Oral steroids may sometimes be beneficial in such setting to decrease the pain and inflammation associated with acute spine pain, but the surgical evaluation should take precedence over all conservative treatment options that may delay resolution of the spinal pathology.
96
Q

A 69-year-old woman underwent surgical resection of a vestibular schwannoma and is recovering in the surgical intensive care unit. On the second day after surgery, the patient complains of a severe headache and neck stiffness. Vital signs reveal blood pressure 140/91 mmHg, heart rate 97 beats/minute, respiratory rate 18 breaths/minute, and temperature 100.9 F. Which of the following is the most likely explanation for the patient’s symptoms?

  1. An inflammatory response to bone dust within the subarachnoid space
  2. Staphylococcus aureus infection
  3. Malignant hyperthermia
  4. Dural sinus thrombosis
A

1. An inflammatory response to bone dust within the subarachnoid space

  • Post-operative meningitis is a feared complication of vestibular schwannoma surgery. Most cases are aseptic and not caused by a microbial infection.
  • Aseptic meningitis is due to an inflammatory reaction against foreign material, such as blood, hemoglobin, and bone dust, within the subarachnoid space.
  • Common presenting symptoms of aseptic meningitis include neck stiffness, fever, headache, and fatigue with negative bacterial cultures.
  • Aseptic and septic meningitis are both associated with cerebrospinal fluid (CSF) leaks, which is the most common postoperative complication of vestibular schwannoma surgery
97
Q

A 24-year-old female patient presents to the emergency department complaining of a headache of 4 weeks duration. She reports the pain is diffuse and has been progressively worsening. She has tried over the counter tylenol and aspirin with minimal effect. A CT of the head revealed a hypodense mass without any enhancement. A biopsy is obtained, which showed a diffuse low- grade astrocytoma. Family history is unknown as she was an orphan. Which of the following would help in establishing the diagnosis?
1. MRI of the liver
2. Colonoscopy
3. CT of the chest
4. Iron panel

A

2. Colonoscopy

  • Patients with Turcot Syndrome can rarely present with an astrocytoma. Given her age, unknown family history and biopsy result, it would be prudent for her to undergo a colonoscopy to rule out Turcot.
  • Diagnosing Turcot syndrome is helpful because making correlations between brain tumors and colorectal cancer can help guide preventative action for either condition. It also provides anticipatory guidance–to know and expect brain vs. colon cancer in patients that exhibit either clinical presentation; early diagnosis leads to early treatment.
  • Turcot syndrome (TS) is the association of primary brain tumors to colorectal cancer.
  • Another correct answer that may have been appropriate for her could have been to check for evidence of APC or MMR gene mutations.
98
Q

A 35-year-old female with a history of adult-onset seizures for three years was referred to the neurosurgery clinic by the neurologist for opinion regarding operative intervention. The patient’s seizure attacks started three years ago, a brain magnetic resonance imaging (MRI) revealed a single left parietal lesion, no surgical intervention was done at that time, the seizure attacks were successfully controlled medically, and there were no focal neurological deficits. Recently the patient kept suffering from seizure attacks despite proper medical therapy by the neurology team. Her most recent studies showed a “popcorn” mixed intensities on brain MRI T1 and T2 sequences with surrounding hemosiderin ring, and a “blooming effect” within the lesion on gradient-echo sequences. No significant increase in size was noted. Noninvasive video-EEG monitoring (VEM) yields positive results consistent with the location of the lesion. After discussing the indications for operative intervention with the neurosurgeon, the patient who had refused the surgical option three years ago is now consenting to surgery. Which of the following is true of the operative considerations for this lesion?

  1. Aim of surgery is to debulk the lesion without removal of the hemosiderin ring
  2. In case of the presence of accompanying venous malformation, surgery should include complete resection of the venous trunk
  3. Dissection should be attempted last on the most eloquent side of the lesion
  4. In operative planning, MRI T2 gradient echo sequence is more accurate in deciding the most superficial location of the lesion than the MRI T1 sequence
A

3. Dissection should be attempted last on the most eloquent side of the lesion

  • In the operative management of Cerebral Cavernous Malformation (CCM), special considerations are crucial for optimum results and avoidance of complications. In the attempt to disect the malformation that is close to eleqouent areas of the brain, proactive techniques are benefecial in minmizing injury: 1. minimize heat exposure by: a. use of irrigating low heat cautery when disecting b. disect the elequent side of the malforamation last and 2. avoidance of retraction, and in case retraction needed it should be minimal and to the least elequent side.
  • Complete removal of the hemosidren ring should the goal to minimize risk of rebleeding and lower seizure recurrence rate.
  • If accompanying developmental venous anomalies (DVA) are present, the trunk of the associated venous malformation should not be removed to avoid cerebral venous infarction.
  • In pre operative planing for the entry of the lesion, MRI T1 is more accurate than the MRI gradient echo, since the “blooming effect” of the lesion on the gradient echo will give a false anticepation of the most superficial site of the malformation
99
Q

A 60-year-old male presents after a stroke that left him with multiple deficits. According to his chart, he has muscle weakness, spasticity, hyperreflexia, and resistance to movement in his right arm and leg. On imaging, it is noted that the ischemic event occurred in the superior division of the affected vessel. What type of aphasia will this patient have?

  1. No aphasia
  2. Global aphasia
  3. Wernicke’s aphasia
  4. Broca’s aphasia
A

4. Broca’s aphasia

  • The middle cerebral artery (MCA) supplies the lateral parts of the cerebral hemispheres. Ischemia results in contralateral paralysis of the arms, legs, and face may be observed due to necrosis of the primary motor cortex, which is observed clinically as muscle weakness, spasticity, hyperreflexia, and resistance to movement (upper motor neuron signs).
  • A dominant, most commonly left-sided, hemisphere stroke results in Broca aphasia if the superior division of the MCA is affected.
  • The middle cerebral artery is the most common, pathologically affected blood vessel overall.
  • In contrast, Wernicke’s or conduction aphasia may be seen if the inferior division of the MCA is affected. A non-dominant, most commonly right-sided, hemisphere stroke results in hemineglect syndrome, presenting with anosognosia, apraxia, and hemispatial neglect.
100
Q

A 25-year-old woman presents to the clinic for evaluation. She has a longstanding history of gait disturbance, developmental delay, and headaches, which are partially relieved with analgesic therapy. The patient states that since birth, she has had a ‘lumpy shoulder blade’ on the right-hand side. However, this causes no discomfort. On chest x-ray, a deformity of the right scapula demonstrating an ‘elevated appearance’ is noted. Which of the following is most likely responsible for the patient’s symptoms and presentation?

  1. Basilar impression
  2. Klippel-Feil syndrome
  3. Pfeiffer syndrome
  4. Myelomeningocele
A

2. Klippel-Feil syndrome

  • Klippel-Feil syndrome is a known association of the Chiari I malformation. Klippel-Feil is commonly associated with a congenital elevation of the scapula, known as a ‘Sprengel deformity’ and can often be seen on the chest radiograph.
  • Chiari I malformation, also called cerebellar tonsillar ectopia, is often an abnormality affecting the hindbrain in isolation.
  • Other associations of Chiari I malformation include syringohydromyelia, hydrocephalus, basilar impression or basilar invagination, occipitalisation of the atlas, unfused posterior arch of C1, platybasia and Klippel-Feil anomaly.
  • Myelomeningocele is not associated with Chiari I but Chiari II malformations.